october 2020 pennsylvania bar examinationquestion no. 1 susan and doug are a married couple who...

81
OCTOBER 2020 PENNSYLVANIA BAR EXAMINATION Essay Questions and Examiners’ Analyses and Performance Test Pennsylvania Board of Law Examiners 601 Commonwealth Avenue, Suite 3600 P.O. Box 62535 Harrisburg, PA 17106-2535 (717) 231-3350 www.pabarexam.org ©2020 Pennsylvania Board of Law Examiners

Upload: others

Post on 01-Aug-2021

4 views

Category:

Documents


0 download

TRANSCRIPT

Page 1: OCTOBER 2020 PENNSYLVANIA BAR EXAMINATIONQuestion No. 1 Susan and Doug are a married couple who reside in Steel Town, Pennsylvania. They have been married for 25 years and have one

OCTOBER 2020

PENNSYLVANIA BAR

EXAMINATION

Essay Questions and Examiners’ Analyses

and

Performance Test

Pennsylvania Board of Law Examiners

601 Commonwealth Avenue, Suite 3600

P.O. Box 62535

Harrisburg, PA 17106-2535

(717) 231-3350

www.pabarexam.org

©2020 Pennsylvania Board of Law Examiners

Page 2: OCTOBER 2020 PENNSYLVANIA BAR EXAMINATIONQuestion No. 1 Susan and Doug are a married couple who reside in Steel Town, Pennsylvania. They have been married for 25 years and have one

i

Table of Contents

Index ..................................................................................................................................................ii

Question No. 1: Facts and Interrogatories, Examiner's Analysis and Grading Guidelines ..............1

Question No. 2: Facts and Interrogatories, Examiner's Analysis and Grading Guidelines ..............9

Question No. 3: Facts and Interrogatories, Examiner's Analysis and Grading Guidelines ..............18

Question No. 4: Facts and Interrogatories, Examiner's Analysis and Grading Guidelines ..............26

Question No. 5: Facts and Interrogatories, Examiner's Analysis and Grading Guidelines ..............35

Question No. 6: Facts and Interrogatories, Examiner's Analysis and Grading Guidelines ..............44

Performance Test and Grading Guidelines ........................................................................................50

Page 3: OCTOBER 2020 PENNSYLVANIA BAR EXAMINATIONQuestion No. 1 Susan and Doug are a married couple who reside in Steel Town, Pennsylvania. They have been married for 25 years and have one

Index

Question No. 1

1. Decedents Estates – Intestate Succession

2. Decedents Estates – Trustee Fiduciary Duty

3. Federal Income Tax – Involuntary Conversion

4. Professional Conduct – Solicitation of Prospective Client

Question No. 2

1. Business Organizations – Shareholder Derivative Action

2. Business Organizations – Court Review of Special Litigation Committee

3. UCC Article II – Non-conforming Goods Buyer’s Rights

4. UCC Article II – Buyer duty to Seller after Rejection of Non-conforming Goods

Question No. 3

1. Criminal Law – First Degree Murder

2. Evidence – Qualification of Expert Witness, Standard for Testimony

3. Evidence – Impeachment with Criminal Convictions

4. Family Law – Jurisdiction and Venue

Question No. 4

1. Constitutional Law – First Amendment Establishment Clause

2. Civil Procedure – Jurisdiction Federal Question & Supplemental

3. Employment Law – Undue Hardship Defense to Title VII Religious Discrimination Claim

Page 4: OCTOBER 2020 PENNSYLVANIA BAR EXAMINATIONQuestion No. 1 Susan and Doug are a married couple who reside in Steel Town, Pennsylvania. They have been married for 25 years and have one

Question No. 5

1(a). Property – Fee Simple Subject to a Condition Subsequent

1(b). Property – Power of Termination/Right of Re-entry

2. Contracts – Third Party Beneficiary

3. Contracts – Unilateral Mistake

4. Property – Severance of Tenancy by the Entireties

Question No. 6

1. Torts – Negligence Premises Liability

2. Pa. Civil Procedure – Venue

3. Evidence – Hypothetical Questions to Expert Witnesses

Page 5: OCTOBER 2020 PENNSYLVANIA BAR EXAMINATIONQuestion No. 1 Susan and Doug are a married couple who reside in Steel Town, Pennsylvania. They have been married for 25 years and have one

Question No. 1

Susan and Doug are a married couple who reside in Steel Town, Pennsylvania. They have

been married for 25 years and have one child together, Sophia, age 21. Sophia is an

undergraduate student at nearby Steel Town University. For the past 20 years, Doug has

worked for Big Cable Co. in Quaker City, Pennsylvania, 300 miles from his home. Doug

primarily worked from home, but he also traveled to Quaker City for a few days each month. In

2002, during business trips to Quaker City, Doug had an extramarital relationship with Maria, a

single coworker. Maria became pregnant. She resigned from her job and moved out of state.

Maria later gave birth to a son, Max. She believed that Doug was Max’s father but never told

Doug.

In March 2019, Doug’s mother Dottie died. Dottie’s will included a gift of $500,000 to a

“Grandchildren’s Trust,” providing as follows:

My trustee shall make distributions from the trust for the undergraduate education

of my grandchildren, including payment of tuition, room and board and other

expenses as each grandchild shall require. For purposes of this trust,

“grandchildren” shall mean the children of my son, Doug. I name Doug as trustee

and, if Doug is unable to serve for any reason, I name Susan as successor trustee.

In August 2019, Max, now 17 years old, enrolled at Quaker City College to begin

undergraduate studies in art history. At the same time, Max contacted Doug and explained that

he was Doug’s son. After an initial shock, Doug agreed to take a genetic test, which was

administered by a physician and proved that Doug was Max’s father. Doug promptly brought

Max back to Steel Town, and introduced Max to Susan, Sophia, and other family and friends as

his long-lost son. He also told Max about the trust that Dottie had established. For the next few

months, Doug maintained contact with Max and saw him regularly.

In October 2019, Max purchased a painting at an antique store for $100. He suspected

that it was actually an original work by a famous artist and had it appraised. The appraiser

confirmed that Max was correct and valued the painting at $25,000. A few weeks later, Max was

1

Page 6: OCTOBER 2020 PENNSYLVANIA BAR EXAMINATIONQuestion No. 1 Susan and Doug are a married couple who reside in Steel Town, Pennsylvania. They have been married for 25 years and have one

scheduled to travel to Steel Town with Doug. He brought the painting with him, planning to

leave it with Doug for safekeeping. On the trip, Doug’s vehicle was involved in an accident with

a semi-tractor-trailer truck. Max sustained serious injuries and Doug was killed.

The painting was destroyed in the crash, and on December 1, 2019, Max received $25,000

from Doug’s insurance company for the loss of the painting. Max used the insurance funds to

finish paying his tuition for the fall 2019 semester.

Soon after Doug’s death, Susan received, in her capacity as trustee, $500,000 from Dottie’s

estate and established a bank account in the name of Grandchildren’s Trust. Susan promptly

paid Sophia’s 2020 tuition using trust funds. A few weeks later, Max asked Susan for trust

money to help with his college expenses for the spring 2020 semester. Susan refused.

1. Assume that Doug died without a will and had a substantial estate and that Susan

will not seek an elective share. Under the Pennsylvania Probate, Estates and

Fiduciaries Code, to whom will Doug’s estate be distributed and in what

proportions?

2. Assume that Max is a valid beneficiary of the Grandchildren’s Trust and that he

files a procedurally proper action seeking to compel Susan to pay his college

expenses from the trust. What arguments should Max make in support of that

action and what is his likelihood of success?

3. What, if any, are the 2019 federal income tax consequences to Max on the receipt of

insurance proceeds? Assume that Max is a cash-basis, calendar-year taxpayer.

Meanwhile, Allen, a Quaker City attorney, learned about the accident. Despite having no

prior contact with Max, he called Max, suggesting that Max retain him for a contingent fee to sue

the truck’s owner for his personal injuries, and said, “Hire me and if we win, I’ll get us both a lot

of money.” Max replied, “I am not interested, don’t contact me again.” Later, Allen sent a follow

up letter, stating, “You should reconsider hiring me, you have a great case!”

4. Did Allen’s communications with Max about the accident violate the Pennsylvania

Rules of Professional Conduct?

2

Page 7: OCTOBER 2020 PENNSYLVANIA BAR EXAMINATIONQuestion No. 1 Susan and Doug are a married couple who reside in Steel Town, Pennsylvania. They have been married for 25 years and have one

Question No. 1: Examiner’s Analysis

1. Under the Pennsylvania Probate, Estates and Fiduciaries (PEF) Code, Doug’s estate will be

governed by the intestacy law, and his estate will pass one-half to Susan and one-quarter

each to Sophia and Max.

Because Doug died without a will, his estate is intestate and governed by the intestacy section of the

PEF Code. “All or any part of the estate of a decedent not effectively disposed of by will” passes

according to the rules of intestate succession.

20 Pa.C.S.A. § 2101; see also In re Estate of Knox, 195 A. 34, 35 (Pa. 1937).

Doug’s estate is governed by intestate succession. Pennsylvania’s intestacy statute provides:

The intestate share of a decedent’s surviving spouse is:

***

(3) If there are surviving issue of the decedent all of whom are issue of the surviving

spouse also, the first $ 30,000 plus one-half of the balance of the intestate estate.

(4) If there are surviving issue of the decedent one or more of whom are not issue of the

surviving spouse, one-half of the intestate estate.

20 Pa.C.S.A. § 2102. “Issue” means the natural or adopted children of the decedent. In re Estate of

Sykes, 383 A.2d 920, 923 (Pa. 1978).

Doug was survived by a spouse, Susan. He was also survived by one child from his marriage to

Susan, Sophia, and another child, Max, whose mother is Maria. Since Doug is survived by a child who

is not Susan’s child, Susan is entitled to one-half of Doug’s estate. The PEF Code provides as follows:

The share of the estate, if any, to which the surviving spouse is not entitled, and the entire

estate if there is no surviving spouse, shall pass in the following order:

(1) Issue. -- To the issue of the decedent.

20 Pa.C.S.A. § 2103. And, as between the decedent’s children:

When the persons entitled to take under this chapter other than as a surviving spouse are

all in the same degree of consanguinity to the decedent, they shall take in equal shares.

20 Pa.C.S.A. § 2104. Natural children of a common parent are, by definition, of the same degree of

consanguinity. Bahl v. Lambert Farms, Inc., 819 A.2d 534, 538 (Pa. 2003).

3

Page 8: OCTOBER 2020 PENNSYLVANIA BAR EXAMINATIONQuestion No. 1 Susan and Doug are a married couple who reside in Steel Town, Pennsylvania. They have been married for 25 years and have one

Doug has two children, Sophia and Max. The intestacy statute specifically addresses children

born to unmarried parents:

Persons born out of wedlock.

***

(c) Child of father. — For purposes of descent by, from and through a person born out

of wedlock, he shall be considered the child of his father when the identity of the father

has been determined in any one of the following ways:

(1) If the parents of a child born out of wedlock shall have married each other.

(2) If during the lifetime of the child, the father openly holds out the child to be his and

receives the child into his home, or openly holds the child out to be his and provides

support for the child which shall be determined by clear and convincing evidence.

(3) If there is clear and convincing evidence that the man was the father of the child,

which may include a prior court determination of paternity.

20 Pa. C.S.A. § 2107. Here, Doug’s identity as Max’s father is established in two ways. First, a

physician administered paternity test has confirmed it. Second, Doug has accepted Max as his

son and held him out to family and friends as his child. Since Doug’s parental status has been

determined under the facts as required by the statute, Max should be treated as Doug’s son and as

an heir to Doug’s intestate estate.

Max and Sophia are both issue of Doug for purposes of intestacy. Because they are of

the same degree of consanguinity – Doug is the father of both – they will take in equal shares.

Susan is entitled to a one-half share as surviving spouse. As surviving issue, Max and

Sophia will split the other one-half share, and each will receive one-quarter of the estate.

2. Max should argue that Susan committed a breach of trust by violating her fiduciary duty

as trustee of the trust, and Max will likely be successful.

The trustee of a trust owes a fiduciary duty to the beneficiaries of the trust. In re Church of St.

James the Less, 888 A.2d 795, 809 (Pa. 2005) citing RESTATEMENT (SECOND) OF TRUSTS § 2 (1959); see

also Sutliff v. Sutliff, 528 A.2d 1318, 1323 (Pa. 1987).

“A violation by a trustee of a duty the trustee owes to a beneficiary is a breach of trust.” 20 Pa.C.S.A. §

7781. If a trustee has breached his or her duty, a court can compel the trustee to perform, among other

remedies. Id.

The PEF Code sets out several specific duties of a trustee as fiduciary.

Duty to administer trust

4

Page 9: OCTOBER 2020 PENNSYLVANIA BAR EXAMINATIONQuestion No. 1 Susan and Doug are a married couple who reside in Steel Town, Pennsylvania. They have been married for 25 years and have one

Upon acceptance of a trusteeship, the trustee shall administer the trust in good faith, in

accordance with its provisions and purposes and the interests of the beneficiaries and in

accordance with applicable law

20 Pa.C.S.A. § 7771. The Grandchildren’s Trust established by Dottie specifically directed the

trustee to make distributions to pay for expenses associated with the undergraduate education of

her grandchildren. By declining to pay Max’s expenses from the trust, Susan has failed to

administer the trust in accordance with its terms and breached her duty to administer the trust.

In addition, the trustee has a duty of loyalty and impartiality.

Duty of loyalty

(a) Duty of trustee. — A trustee shall administer the trust solely in the interests of the

beneficiaries.

20 Pa.C.S.A. § 7772.

Impartiality

If a trust has two or more beneficiaries, the trustee shall act impartially in investing,

managing and distributing the trust property, giving due regard to the beneficiaries’

respective interests in light of the purposes of the trust. The duty to act impartially does

not mean that the trustee must treat the beneficiaries equally. Rather, the trustee must

treat the beneficiaries equitably in light of the purposes of the trust.

20 Pa.C.S.A. § 7773.

The Pennsylvania Supreme Court has adopted the approach of the Restatement (Second) of

Trusts on the requirement of impartiality. In re Estate of Hamill, 410 A.2d 770, 773 (Pa. 1980); In re

Estate of Sewell, 409 A.2d 401, 402 (Pa. 1979). “When there are two or more beneficiaries of a trust,

the trustee is under a duty to deal impartially with them.” RESTATEMENT (SECOND) OF TRUSTS § 183

(Am. Law Inst. 1959).

In Sewell, the settlor had established a trust providing that income was to be distributed first to

his wife and, after her death, to his children. In the event a child predeceased his wife, then to that

child’s children. In re Estate of Sewell, 409 A.2d at 402. The settlor had two children. The child who

predeceased the settlor’s wife had an adopted daughter. Id. Despite having knowledge of this

grandchild of the settlor, the trustee failed to make income distributions to her. Id. at 402-03. The court

held that the trustee breached her fiduciary duty to deal impartially with the beneficiaries. Id. at 402.

Similar to Sewell, Susan refused to make distributions to Max despite the fact that Max was a

grandchild of the settlor of the trust and was clearly entitled to distributions under the dispositive

provisions of the trust.

5

Page 10: OCTOBER 2020 PENNSYLVANIA BAR EXAMINATIONQuestion No. 1 Susan and Doug are a married couple who reside in Steel Town, Pennsylvania. They have been married for 25 years and have one

Susan had a fiduciary obligation to treat Sophia and Max impartially. Instead, she favored her

own daughter, to the exclusion of Max. Susan has breached her duty as trustee and Max will likely be

successful in his action seeking payment of tuition.

3. The federal income tax consequences to Max on receipt of the insurance proceeds would be

a realized gain of $24,900 that Max would have to report as income on his 2019 federal

income taxes.

Under the Internal Revenue Code, funds received from an involuntary conversion of property

constitute taxable income in certain circumstances.

(a) General rule. -- If property (as a result of its destruction in whole or in part, theft,

seizure, or requisition or condemnation or threat or imminence thereof) is compulsorily

or involuntarily converted—

(1) Conversion into similar property.-- Into property similar or related in service or use

to the property so converted, no gain shall be recognized.

(2) Conversion Into Money.-- Into money or into property not similar or related in

service or use to the converted property, the gain (if any) shall be recognized except

to the extent hereinafter provided in this paragraph . . . .

26 U.S.C.S. § 1033. If the taxpayer uses the money received to replace the converted property

with similar property, there is no recognition of the gain. 26 U.S.C.S. § 1033(a)(1).

Here, Max’s painting was destroyed in the car crash. He received $25,000 in insurance

proceeds. He did not use the funds to purchase similar property, he used them to pay his college

tuition. Therefore, Max must treat the conversion of the painting into money as a taxable event

and recognize any gain.

Gross income includes “[g]ains derived from dealings in property.” 26 U.S.C.S. § 61(a)(3).

“The amount realized from the sale or other disposition of property shall be the sum of any money

received. . . .” 26 U.S.C.S. § 1001(b) “The gain from the sale or other disposition of property shall be

the excess of the amount realized therefrom over the adjusted basis. . . .” 26 U.S.C.S. § 1001(a). The

basis of property is the cost of the property. See 26 U.S.C.S. §§ 1011; 1012(a).1

Max paid $100 for the painting. His basis is $100. He received $25,000 for the painting, based

on its appraised value. His amount realized, less his basis, results in a gain of $24,900 that Max must

report as income on his taxes.

4. Allen’s contacts with Max violated the Pennsylvania Rule of Professional Conduct

governing solicitation of clients.

1 Adjustments to basis are required to be made for certain capital expenditures, i.e. depreciation and other items, none of

which are applicable to the instant facts. See 26 U.S.C.S. § 1016.

6

Page 11: OCTOBER 2020 PENNSYLVANIA BAR EXAMINATIONQuestion No. 1 Susan and Doug are a married couple who reside in Steel Town, Pennsylvania. They have been married for 25 years and have one

The Pennsylvania Rules of Professional Conduct govern solicitation of prospective clients, and

provides, in relevant part, as follows:

A lawyer shall not solicit in-person or by intermediary professional employment from a

person with whom the lawyer has no family or prior professional relationship when a

significant motive for the lawyer's doing so is the lawyer's pecuniary gain, unless the

person contacted is a lawyer or has a family, close personal, or prior professional

relationship with the lawyer.

Pa. R.P.C. 7.3(a). The definition of “solicit” includes telephone communications. Id. Therefore, Allen’s

first contact with Max, by telephone, is governed by this rule. Allen had no prior relationship with Max.

His comments about recovering “a lot of money for both of us” indicate that a significant motive for

contacting Max was Allen’s own financial gain. As such, it is likely that Allen’s initial contact with

Max violated Rule 7.3(a).

Max’s response to this contact was to decline and ask Allen not to contact him again.

Nonetheless, Allen sent a follow-up letter again encouraging Max to file a lawsuit. Rule 7.3 further

provides as follows:

(b) A lawyer may contact, or send a written communication to, the target of the

solicitation for the purpose of obtaining professional employment unless:

***

(2) the person has made known to the lawyer a desire not to receive communications

from the lawyer . . . .

Pa. R.P.C. 7.3(b).

A solicitation is a targeted communication initiated by the lawyer that is directed to a

specific person and that offers to provide, or can reasonably be understood as offering to

provide, legal services.

Pa. R.P.C. 7.3 cmt. 1. Allen’s letter is a solicitation. He is offering to provide legal services in the

nature of a suit for damages arising from personal injuries sustained in the accident. While written

communications are generally acceptable, this one violates Rule 7.3(b) because Max has specifically

asked Allen not to contact him again.

In summary, both of Allen’s contacts with Max violate the Pennsylvania Rules of Professional

Conduct.

7

Page 12: OCTOBER 2020 PENNSYLVANIA BAR EXAMINATIONQuestion No. 1 Susan and Doug are a married couple who reside in Steel Town, Pennsylvania. They have been married for 25 years and have one

Question No. 1: Grading Guidelines

1. Intestate succession.

Comments: Candidates should recognize that in the absence of a will, distribution of a decedent’s estate

is governed by the intestacy statute and apply the statute to determine the appropriate beneficiaries and

the share each would receive.

5 points.

2. Fiduciary duty of trustee.

Comments: Candidates should identify and discuss a trustee’s duty to administer the trust according to

its terms and to treat the beneficiaries of the trust impartially.

5 points.

3. Involuntary conversion.

Comments: Candidates should identify and discuss the tax consequences of an involuntary conversion

of property and determine the taxpayer’s gain or loss for the destroyed painting.

5 points.

4. Solicitation of prospective clients.

Comments: Candidates should identify and apply the Pennsylvania Rule of Professional Conduct

regarding solicitation of clients.

5 points.

8

Page 13: OCTOBER 2020 PENNSYLVANIA BAR EXAMINATIONQuestion No. 1 Susan and Doug are a married couple who reside in Steel Town, Pennsylvania. They have been married for 25 years and have one

Question No. 2

Wood, Inc. (Wood), a Pennsylvania corporation, operates a sawmill, lumber

yard, and wood finishing plant in northwest Pennsylvania. Wood has historically

sold finished lumber, lumber specially treated for outdoor use, and bulk and

bundled firewood.

Al, Ben, Chris, Dee, and Ed each inherited one-fifth of Wood’s issued and

outstanding stock from their mother in 2010. Al, Ben, and Ed have been Wood’s

directors and officers since 2010. Chris and Dee live in Florida and never worked at

Wood.

Last year, with the aid of Wood’s CPA, Ben analyzed Wood’s firewood sales

and concluded that Wood could be more profitable if it were not in the firewood

business. Ben presented the analysis to Wood’s board. Ben told the board that

getting out of the firewood business would result in Wood losing revenue for several

years but would ultimately lead to increased profits as Wood converted firewood

processing to other functions. Ben made a motion that Wood cease selling firewood

and shut down its firewood operations. The board reviewed and accepted Ben’s

analysis, approved the motion, and immediately terminated Wood’s firewood

operations. To date, as Ben had predicted, his plan has negatively affected Wood’s

profit margin, reducing profits and distributions to Wood’s shareholders.

Last week, a former Wood bookkeeper whom Ben had fired called Chris and

told her about the board’s action to terminate the firewood operations and the

negative effect that it was having on profits. Chris called Al and Ben to discuss the

9

Page 14: OCTOBER 2020 PENNSYLVANIA BAR EXAMINATIONQuestion No. 1 Susan and Doug are a married couple who reside in Steel Town, Pennsylvania. They have been married for 25 years and have one

decision. They told Chris that the decision was in Wood’s best interests and that

profits would be down for a year or two, but soon Wood would see profits increase

due to the board’s decision. Chris yelled, “You have failed to act in the best interests

of Wood and its shareholders.” Chris demanded the board rescind its action and

make Wood whole for lost profits. Chris also stated, “I’m done with you two. I will

see you in court.”

Sheds, Inc. (Sheds) builds and sells outdoor storage sheds in southeast

Pennsylvania. Sheds was searching for a new supplier of specially-treated lumber

suitable for outdoor use (treated lumber) and found Wood on the internet. Sheds

contacted Al, who told Sheds that Wood had not sold in Sheds’ area and had no

office or agent there, but would still be interested in selling to Sheds. Last month,

Sheds sent a written purchase order for $20,000 worth of treated lumber to Wood.

The purchase order specifically required all of the lumber to be treated lumber.

Wood accepted the purchase order forming a valid and enforceable contract

(contract) that required delivery of the treated lumber by today. The contract does

not contain limitation-of-remedies provisions.

Yesterday, Wood’s shipment of lumber arrived at Sheds right before the time

Sheds’ closed for the day. Sheds promptly inspected the delivery and discovered

40% of the lumber was not specially treated lumber suitable for outdoor use; thus, it

did not conform to the requirements of the contract. Sheds has no use for the

untreated lumber.

1. Chris has retained counsel to file a shareholder derivative action

against Wood’s board. Under the Pennsylvania Business Corporations

10

Page 15: OCTOBER 2020 PENNSYLVANIA BAR EXAMINATIONQuestion No. 1 Susan and Doug are a married couple who reside in Steel Town, Pennsylvania. They have been married for 25 years and have one

Law, what facts arguably support the filing of a derivative action by

Chris and what procedural requirements would Chris have to meet

prior to filing the action?

2. Assume that Chris filed a derivative action. In response, Wood’s board

appointed a special litigation committee to review the derivative

action. The special litigation committee consists of three independent

disinterested individuals capable of exercising objective judgment. The

special litigation committee, after extensive review of all relevant facts,

determines the suit should be dismissed. Assuming the board has met

all procedural and notice requirements, under the Pennsylvania

Business Corporations Law, what is the scope of the court’s review of

the special litigation committee action and what is the likely result of

that review?

3. Under the Pennsylvania Uniform Commercial Code (Code) what rights

does Sheds have as the buyer of the lumber given the non-conforming

delivery by Wood, and, what would be the best way for Sheds to

proceed to protect and exercise its rights?

4. Under the Code what duties, if any, does Sheds, as a merchant

buyer, have to Wood regarding the lumber if any or all of it is

not accepted by Sheds?

11

Page 16: OCTOBER 2020 PENNSYLVANIA BAR EXAMINATIONQuestion No. 1 Susan and Doug are a married couple who reside in Steel Town, Pennsylvania. They have been married for 25 years and have one

Question No. 2: Examiner’s Analysis

1. Chris should file the derivative action and argue that the board breached its

fiduciary duty to Wood evidenced by the drop in profits directly related to the

board’s action of terminating Wood’s firewood operations. Before filing the action,

Chris should demand, in writing, that Wood or its board file or authorize a suit

against the directors to recover Wood’s lost profits and to rescind the firewood

decision.

A shareholder’s derivative action is “an action to enforce a secondary right brought by

one or more stockholders or members of a corporation or similar entity because the corporation

or entity refuses or fails to enforce rights which could be asserted by it.” Pa.R.C.P. No. 1506(a).

The right to bring the suit is a “secondary” right because it is an equitable action brought by a

shareholder to enforce a right of the corporation itself and not a personal, or primary, right of the

shareholder. Derivative suits developed as a way for shareholders to protect the interests of the

corporation from the malfeasance or misfeasance of its directors. A shareholder’s derivative

action is filed on behalf of the corporation and not to enforce a direct claim held by the

shareholder. See 3 W. EDWARD SELL & WILLIAM H. CLARK, JR., Pennsylvania Business

Corporations, § 1782.2 (Rev. 2d ed. 1998). Here, the derivative action that Chris files will

benefit more than just Chris; i.e., it will benefit the corporation and its shareholders generally,

and, therefore, is appropriate if a basis for filing exists.

The Pennsylvania Business Corporations Law of 1988 (BCL) provides, in relevant part,

as follows:

A director of a business corporation shall stand in a fiduciary relation to the

corporation and shall perform his duties as a director . . . in good faith, in a

manner he reasonably believes to be in the best interests of the corporation and

with such care, including reasonable inquiry, skill and diligence, as a person of

ordinary prudence would use under similar circumstances.

15 Pa.C.S.A. § 1712(a). A similar standard applies to officers. See 15 Pa.C.S.A. § 1712(c).

“Underlying any discussion of fiduciary duty is the concept of ‘good faith.’ Good faith may be

defined as behavior that is honest and without fraud, delusion or deceit; the concept always

incorporates the meaning of ‘pure heartedness.’” SELL & CLARK, supra at § 1712.4. In

evaluating the actions of a director to determine if he has acted in good faith one must consider

both the facts and whether or not the director believed that he was acting in the best interests of

the corporation. Id.

In this case, Chris believes that there has been harm to the corporation because of action

taken by the directors. “The derivative action provides corporate shareholders the means to

assert a cause of action arising from a wrong to the corporation.” JOHN W. MCLAMB, JR. AND

WENDY C. SHIBA, Pennsylvania Corporation Law & Practice, § 6.2a. (1994). It is undisputed

that the decision to terminate firewood operations has resulted in diminished profits for the

corporation and its shareholders. This harm provides an arguable basis for Chris to proceed with

12

Page 17: OCTOBER 2020 PENNSYLVANIA BAR EXAMINATIONQuestion No. 1 Susan and Doug are a married couple who reside in Steel Town, Pennsylvania. They have been married for 25 years and have one

the derivative action even though the claim and action may ultimately be unsuccessful as there

appears to have been a rational basis for the decision.

Assuming that an arguable basis exists, a shareholder’s derivative suit may be filed and

maintained in accordance with Subchapter F of the BCL and Pa.R.C.P. No. 1506. 15 Pa.C.S.A.

§§ 1781-1784. Subchapter F provides, subject to certain exceptions and conditions dealing with

non-shareholder plaintiffs and security that may be required to be posted not applicable under the

above facts, the plaintiff filing a derivative action must have been a shareholder at the time of the

transaction giving rise to the suit and also must be a current shareholder. 15 Pa.C.S.A. §

1782(a); Pa.R.C.P. No. 1506(a)(1). The facts demonstrate that Chris meets these requirements.

She was a shareholder when the board terminated its firewood business and continues to be a

shareholder. She should file the derivative action and argue that the board breached its fiduciary

duty to act in Wood’s interests by eliminating the sale of firewood consequently reducing

Wood’s profits.

Also, unless excused, prior to filing the derivative action the plaintiff must have made a

demand on the corporation or the board of directors of the corporation requesting that it cause the

corporation to bring an action to enforce the rights asserted by the plaintiff on behalf of the

corporation. 15 Pa.C.S.A. § 1781(a)(1). The demand “must be in record form and give notice

with reasonable specificity of the essential facts relied upon to support each of the claims made

in the demand.” 15 Pa.C.S.A. § 1781(c). “Record form” means a form “[i]nscribed on a tangible

medium or stored in an electronic or other medium and retrievable in perceivable form.” 15

Pa.C.S.A. § 102(a). This demand requirement is excused “only if the plaintiff makes a specific

showing that immediate and irreparable harm to the business corporation would otherwise

result.” 15 Pa.C.S.A. § 1781(b)(1). Pa.R.C.P. No. 1506(2) similarly requires that the plaintiff

set forth in its complaint the efforts made by the plaintiff to secure enforcement by the

corporation.

In this case, Chris can satisfy the demand requirement in section 1781 by sending a letter

or electronic communication to the board requesting it take action on behalf of the corporation

regarding its decision for Wood to cease selling firewood. Even though the board is the entity

that made the decision, the demand requirement would not be excused because the facts do not

provide Wood would suffer immediate and irreparable harm if the demand was made. Chris can

satisfy the requirements of rule 1506(2) by pleading the necessary facts showing she made a

written demand to the board in the body of her derivative action complaint. See 15 Pa.C.S.A. §

1781.

2. The court’s scope of review is to determine if the members of the special litigation

committee (SLC) met the qualifications required by the BCL and if its

recommendation was made in good faith, independently, and with reasonable care.

If the court concludes that the SLC met these standards, then the court should

approve the determination of the SLC and grant the board’s motion to dismiss the

derivative action suit.

13

Page 18: OCTOBER 2020 PENNSYLVANIA BAR EXAMINATIONQuestion No. 1 Susan and Doug are a married couple who reside in Steel Town, Pennsylvania. They have been married for 25 years and have one

If the corporation appoints an SLC and does not authorize the plaintiff to proceed with

the derivative action the matter shall proceed in accordance with section 1783 of the BCL.

Section 1783 sets forth its general rule as follows:

If a business corporation or the board of directors receives a demand to bring an

action to enforce a right of the corporation, or if a derivative action is commenced

before demand has been made on the corporation or the board, the board may

appoint a special litigation committee to investigate the claims asserted in the

demand or action and to determine on behalf of the corporation or recommend to

the board whether pursuing any of the claims asserted is in the best interests of the

corporation. The corporation shall send a notice in record form to the plaintiff

promptly after the appointment of a committee under this section notifying the

plaintiff that a committee has been appointed and identifying by name the

members of the committee. A committee may not be appointed under this section

if every shareholder of the corporation is also a director of the corporation.

15 Pa.C.S.A. § 1783(a). Pursuant to Section 1783, a majority of the directors not named as

actual or potential parties in the demand or action or, if all of the directors are so named, a

majority of those directors so named can appoint the SLC. 15 Pa.C.S.A. § 1783(d).

Additionally, section 1783 requires that the SLC shall be comprised of two or more individuals

who need not be shareholders or directors and who “(1) are not interested in the claims asserted

in the demand or action, [and] (2) are capable as a group of objective judgment in the

circumstances.” 15 Pa.C.S.A. § 1783(c).

If a derivative action is commenced before a determination is made by the SLC, the

corporation is required to file with the court “a statement of the determination whenever made

and a report of the committee supporting the determination.” 15 Pa.C.S.A. § 1783(f)(1). The

statement of determination and report must be served on all parties. Id. The corporation must

also file a motion, pleading, or consent with the court consistent with the determination. Id.

Section 1783 further provides:

If the determination is one described in subsection (e) . . . (7) [(e)(7) is a

determination that an action already commenced be dismissed], the court shall

determine whether the members of the committee met the qualifications required

under subsection (c)(1) and (2) and whether the committee conducted its

investigation and made its recommendation in good faith, independently and with

reasonable care. If the court finds that the members of the committee met the

qualifications required under subsection (c)(1) and (2) and that the committee

acted in good faith, independently and with reasonable care, the court shall

enforce the determination of the committee.

15 Pa.C.S.A. § 1783(f)(3). It should be noted that the committee drafters of section 1783 stated,

“If a court determines that the members of the committee met the qualifications required under

subsection (c)(1) and (2) and that the committee conducted its investigation and made its

recommendation in good faith, independently and with reasonable care, it makes no sense to

14

Page 19: OCTOBER 2020 PENNSYLVANIA BAR EXAMINATIONQuestion No. 1 Susan and Doug are a married couple who reside in Steel Town, Pennsylvania. They have been married for 25 years and have one

substitute the court’s legal judgment for the business judgment of the committee.” 15 Pa.C.S.A.

§ 1783 comm. cmt. (2016).

The facts indicate that the SLC was comprised of three disinterested, independent

individuals capable of exercising objective judgment. If the SLC reviewed the analysis prepared

by Ben and Woods’ CPA, considered the fact that the board likewise reviewed the analysis, and

concluded that Woods’ best interests were served by terminating firewood operations, the SLC

would have had a legitimate basis to conclude that the board had acted properly and not in

violation of its duty of care to the corporation and its shareholders. If the court concludes that

the SLC met the standards of section 1783, then the court should accept the findings and

determination of the SLC and dismiss the derivative suit.

3. Sheds should take no action that evidences an acceptance of the lumber. It has the

right to declare the shipment to be nonconforming and to notify Wood that it is

rejecting the whole shipment or accepting any commercial unit or units and

rejecting the remainder of the shipment.

Article II of the Pennsylvania Uniform Commercial Code on Sales (Code) is generally

applicable to transactions in goods. 13 Pa.C.S.A. § 2102. “Goods” are “all things . . . which are

movable at the time of identification to the contract for sale . . . .” 13 Pa.C.S.A. § 2105(a). The

lumber that was the subject matter of the contract is movable. Therefore, the contract is subject

to the provisions of the Code.

Wood has failed to provide Sheds with lumber that conformed to the requirements of its

contract. The contract required all of the lumber to be specially-treated lumber for outdoor use

and 40% of the lumber that was delivered was untreated. This is a failure to comply with the

terms of the contract. Thus, Wood has breached the terms of the contract. As a result, so long as

an acceptance has not occurred on Sheds’ part, Sheds can advise Wood of the nonconformity and

exercise its rights under the Code to reject all or part of the goods delivered.

Section 2601 of the Code provides, “Subject to the provisions of this division on breach

in installment contracts (section 2612) and unless otherwise agreed under the sections on

contractual limitations of remedy (sections 2718 and 2719), if the goods or the tender of delivery

fail in any respect to conform to the contract, the buyer may: (1) reject the whole; (2) accept the

whole; or (3) accept any commercial unit or units and reject the rest.” 13 Pa.C.S.A. § 2601. The

contract between Wood and Sheds is not an installment contract and their contract did not

address limitation of remedies (i.e., there is no contract term prohibiting rejection). Therefore, it

would appear that Sheds has the rights of rejection set forth in Section 2601.

If, however, a buyer has accepted the goods, then the buyer cannot reject the goods. See

JAMES J. WHITE AND ROBERT S. SUMMERS, Uniform Commercial Code, § 8-3(a) (4th ed. 1995).

The Code provides that acceptance occurs when the buyer, after a reasonable opportunity to

inspect the goods, either signifies to the seller that the goods are conforming or that the buyer

will take them despite a nonconformity; or the buyer, after a reasonable time to inspect, fails to

reject the goods; or the buyer does an act inconsistent with the ownership of the seller. See 13

15

Page 20: OCTOBER 2020 PENNSYLVANIA BAR EXAMINATIONQuestion No. 1 Susan and Doug are a married couple who reside in Steel Town, Pennsylvania. They have been married for 25 years and have one

Pa.C.S.A. § 2606. Sheds has just discovered the nonconformity and has taken no action

signifying acceptance. Accordingly, acceptance has not occurred.

Section 2602 of the Code provides that “[r]ejection of goods must be within a reasonable

time after their delivery or tender. It is ineffective unless the buyer seasonably notifies the

seller.” 13 Pa.C.S.A. § 2602(a). “Reasonable time . . . depends on the nature, purpose and

circumstances” and “seasonableness” is “within the time agreed or, if no time is agreed, at or

within a reasonable time.” 13 Pa.C.S.A. § 1205(a), (b). Thus, if Sheds wishes to reject the

goods, Sheds must give notice of the rejection within a reasonable time of delivery.

Additionally, section 2605 of the Code states, “The failure of the buyer to state in connection

with rejection a particular defect which is ascertainable by reasonable inspection precludes him

from relying on the unstated defect to justify rejection or to establish breach: (1) where the seller

could have cured it if stated seasonably; or (2) between merchants when the seller has after

rejection made a request in writing for a full and final written statement of all defects on which

the buyer proposes to rely.” 13 Pa.C.S.A. § 2605(a). Based on the Code, the rejection notice by

Sheds in the instant case should set forth specifically the nonconformity that supports the

rejection to avoid an argument of non-compliance with the requirements of Section 2605.

Accordingly, if Sheds wishes to reject the lumber due to nonconformity with the contract

provisions it should promptly notify Wood that it is rejecting the lumber because it did not

conform to the specifications set forth in the contract; i.e., that all of the lumber be specially-

treated for outdoor use.

Thus, Sheds may, provided it complies with the notice requirements of the Code, advise

Wood that it is either rejecting the entire shipment or accepting only the conforming lumber and

rejecting the remainder.

4. Sheds will be relieved of its obligation to pay for the lumber that it rejects but must

hold the lumber for the benefit of Wood and follow reasonable instructions from

Wood regarding disposition of the rejected lumber.

Upon a valid rejection of goods, a buyer is relieved of its duty to pay for the goods. 13

Pa.C.S.A. § 2711. Thus, Sheds will not need to pay Wood for any lumber that it rejects.

Section 2603 of the Code provides the following:

Duties of merchant buyer as to rightfully rejected goods

(a) General rule.--Subject to any security interest in the buyer (section 2711(c)),

when the seller has no agent or place of business at the market of rejection a

merchant buyer is under a duty after rejection of goods in his possession or

control to follow any reasonable instructions received from the seller with respect

to the goods and in the absence of such instructions to make reasonable efforts to

sell them for the account of the seller if they are perishable or threaten to decline

in value speedily. Instructions are not reasonable if on demand indemnity for

expenses is not forthcoming.

16

Page 21: OCTOBER 2020 PENNSYLVANIA BAR EXAMINATIONQuestion No. 1 Susan and Doug are a married couple who reside in Steel Town, Pennsylvania. They have been married for 25 years and have one

(b) Reimbursement for expenses and commission.--When the buyer sells goods

under subsection (a), he is entitled to reimbursement from the seller or out of the

proceeds for reasonable expenses of caring for and selling them, and if the

expenses include no selling commission then to such commission as is usual in

the trade or if there is none to a reasonable sum not exceeding 10% on the gross

proceeds.

(c) Good faith conduct.--In complying with this section the buyer is held only to

good faith and good faith conduct under this section is neither acceptance nor

conversion nor the basis of an action for damages.

13 Pa.C.S.A. § 2603.

Based on these Code sections, the buyer has an obligation to hold the goods for a time

sufficient for the seller to remove them under section 2602(b)(2) unless “(1) the seller has no

agent or place of business at the market of rejection; (2) the buyer is a merchant; and (3) the

goods are in the buyer’s possession and control[,]” in which case section 2603 is applicable. See

JAMES J. WHITE AND ROBERT S. SUMMERS, Uniform Commercial Code, § 8-3(d), (4th ed. 1995).

Section 2603 applies here because Wood and Sheds are at opposite ends of the state and

Sheds is located in an area where Wood does not normally operate. Sheds is a merchant as

defined in 13 Pa.C.S.A. § 2104 and is clearly in possession of the goods. Pursuant to section

2603, Sheds must hold the lumber and await reasonable instructions from Wood as to reshipping,

storing, delivery to a third party, reselling, or the like. See 13 Pa.C.S.A. § 2603 cmt. 1.

17

Page 22: OCTOBER 2020 PENNSYLVANIA BAR EXAMINATIONQuestion No. 1 Susan and Doug are a married couple who reside in Steel Town, Pennsylvania. They have been married for 25 years and have one

Question No. 3

Mary Beth and Brian have been married for six years and have continuously resided in C

County, Pennsylvania. On November 1, 2019, Mary Beth’s friend, Louise, permanently separated from

her husband, Frank, and relocated from Ohio to C County, Pennsylvania, where Louise intended to

permanently reside. Frank remained in Ohio where he intended to reside permanently.

Mary Beth had become increasingly unhappy in her marriage and decided in early March 2020

to come up with a plan to permanently get Brian out of her life. On March 14, Mary Beth purchased a

$1,000,000 life insurance policy on Brian’s life and named herself the sole beneficiary. A few days

later, she went to a local pharmacy and told the pharmacist that she wanted rat poison because she had

an infestation of rats at her home. Mary Beth’s true intention was to purchase a product containing

arsenic, which she knew from her recent internet research was an ingredient of rat poison that was fatal

to humans in high doses. The pharmacist recommended a rat poison that did contain high amounts of

arsenic, and he told Mary Beth that “this will kill anything.” In response, Mary Beth asked if it was

enough to kill a human and the pharmacist said yes. Mary Beth purchased the rat poison. Over the next

two weeks, Mary Beth mixed the poison into the food that she served to Brian. As a result, Brian

became increasingly sick with vomiting and severe stomach pain and died on March 30, 2020.

Two days after Brian’s death, Rachel performed an autopsy on Brian’s body. Rachel is a board-

certified forensic pathologist with twenty years of forensic pathology experience. As such, she

possesses the requisite training to rule upon the effects of poisons, including arsenic, on the human

body. At the autopsy, Rachel took tissue samples from Brian’s body. Rachel subsequently tested the

samples and eventually concluded that Brian had a concentration of arsenic in his body that was ten

times the fatal limit for human beings. Based upon all of her work, which was performed using

generally accepted methods in the field of forensic pathology, as well as the results of the police

Page 23: OCTOBER 2020 PENNSYLVANIA BAR EXAMINATIONQuestion No. 1 Susan and Doug are a married couple who reside in Steel Town, Pennsylvania. They have been married for 25 years and have one

investigation, Rachel concluded that Brian’s cause of death was arsenic poisoning and the manner of

death was homicide.

As part of its investigation, a C County police detective interviewed Louise, who told the

detective that, a week after Brian’s death, Mary Beth told Louise that she had to kill Brian because she

couldn’t live with him any longer. The detective ran a criminal record check on Louise and learned that

she had convictions in C County for robbery in 2014, DUI in 2015, and burglary in June 2005. As part

of required pre-trial disclosures, the Commonwealth turned over all necessary paperwork regarding

Louise’s three convictions to Mary Beth’s attorney. The detective also spoke with the pharmacist, who

relayed his conversation with Mary Beth about the lethal nature of the rat poison. The detective also

determined that Mary Beth’s home had no evidence of rat infestation; however, he found, pursuant to a

properly executed search warrant, a near empty box of rat poison hidden in Mary Beth’s closet. The

detective ruled out any other way Brian was exposed to arsenic that could have contributed to his death.

1. Based on the above facts, what is the most serious homicide charge that should be filed

against Mary Beth for Brian’s death and with what likely result?

2. Assume Mary Beth is tried for the above homicide charge in the Court of Common Pleas

of C County and the Commonwealth calls Rachel as an expert in the field of forensic

pathology to testify as to the cause and manner of Brian’s death. What must the

Commonwealth establish to successfully qualify and have Rachel testify as an expert in

the field of forensic pathology and to what level of certainty must Rachel’s expert

opinion be based?

3. Assume Louise testifies at Mary Beth’s homicide trial. Under the Pennsylvania rules of

evidence, would any of the above three listed crimes be admissible for impeachment

purposes provided the defense complies with any applicable notice requirements?

4. If Louise wants to file a divorce action against Frank in Pennsylvania, has jurisdiction

been established for her to do so and in which Pennsylvania county(ies) can the divorce

action be filed?

Page 24: OCTOBER 2020 PENNSYLVANIA BAR EXAMINATIONQuestion No. 1 Susan and Doug are a married couple who reside in Steel Town, Pennsylvania. They have been married for 25 years and have one

Question No. 3: Examiner’s Analysis

1. The most serious homicide charge that should be filed against Mary Beth is first-degree

murder which will likely be supported by the facts.

“A criminal homicide constitutes murder of the first degree when it is committed by an

intentional killing.” 18 Pa. C.S.A. § 2502(a). An intentional killing is defined as “[k]illing by means of

poison, or by lying in wait, or by any other kind of willful, deliberate and premeditated killing.” 18 Pa.

C.S.A. §2502(d). “There are three elements of first-degree murder: (1) a human being was unlawfully

killed; (2) the defendant was responsible for the killing; and (3) the defendant acted with malice and a

specific intent to kill.” Commonwealth v. Jordan, 65 A.3d 318, 323 (Pa. 2013) citing Commonwealth v.

Houser, 18 A.3d 1128, 1133 (Pa. 2011). “Malice is defined as a ‘wickedness of disposition, hardness of

heart, cruelty, recklessness of consequences, and a mind regardless of social duty.’” Commonwealth v.

Rementer, 598 A.2d 1300, 1308 (Pa. Super. 1991) (quoting Commonwealth v. Pigg, 571 A.2d 438, 443

(1990), appeal denied, 581 A.2d 571 (Pa. 1991)). “Malice may be inferred from all of the circumstances

surrounding the conduct of the accused.” Rementer, 598 A.2d at 1308 (citing Commonwealth v. Petrino,

480 A.2d 1160 (Pa. Super. 1974)). In a prosecution for murder by poisoning, “[t]he Commonwealth

must show beyond reasonable doubt that [the victim’s] death resulted from poison feloniously

administered.” Commonwealth v. Holt, 39 A.2d 372, 377 (Pa. 1944). “[W]hile demonstrating motive is

not necessary to prove murder in the first degree, it is probative evidence of specific intent to kill.”

Commonwealth v. Tempest, 432 A.2d 952, 955 (Pa. 1981) (citing Commonwealth v. Tomoney, 412 A.2d

531, 534 (Pa. 1980)).

The facts provide that in early March, 2020, Mary Beth came up with a plan to permanently get

Brian out of her life. Shortly thereafter, she purchased a $1,000,000 life insurance policy on Brian’s life

and named herself as sole beneficiary. A few days later, she went to the local pharmacy to purchase rat

poison that she knew from her research contained arsenic and was fatal to humans in high doses.

Through her conversations with the pharmacist, she confirmed that the product that she was purchasing

would kill a human being. In the following two weeks, she mixed the rat poison with the food that she

served to Brian. The facts indicate that Brian became increasingly sick and died on March 30, 2020.

The forensic pathologist concluded that Brian’s death was caused by the arsenic poisoning. Based upon

the above facts, it is clear that Mary Beth committed an intentional killing of Brian. She took multiple

steps from researching and then purchasing the rat poison for a nonexistent rat problem to putting the

poison in Brian’s food, which led to his death. All of these actions establish her intent to kill Brian

through a premeditated plan. Mary Beth’s actions, when viewed in their totality amply support the

malice requirement for first-degree murder. The fact that she purchased a $1,000,000 life insurance

policy supplies an additional motive to kill Brian. Mary Beth’s admission to Louise that she killed Brian

also provides proof of the first-degree murder charge.

In conclusion, the most serious homicide charge that should be filed against Mary Beth is first

degree murder and the Commonwealth will likely be successful in pursuing this charge.

2. The Commonwealth must qualify Rachel as an expert based upon her training and

experience and show that her testimony is necessary to assist the jury in understanding the

medical evidence and determining the facts surrounding Brian’s death. Rachel must

Page 25: OCTOBER 2020 PENNSYLVANIA BAR EXAMINATIONQuestion No. 1 Susan and Doug are a married couple who reside in Steel Town, Pennsylvania. They have been married for 25 years and have one

render her conclusions within a reasonable degree of certainty in the field of forensic

pathology.

Pa.R.E. 702 provides as follows:

A witness who is qualified as an expert by knowledge, skill, experience, training, or

education may testify in the form of an opinion or otherwise if:

(a) the expert’s scientific, technical, or other specialized knowledge is beyond that

possessed by the average layperson;

(b) the expert’s scientific, technical, or other specialized knowledge will help the trier

of fact to understand the evidence or to determine a fact in issue; and

(c) the expert’s methodology is generally accepted in the relevant field.1

It is well established in this Commonwealth that the standard for qualification of an

expert witness is a liberal one. The test to be applied when qualifying an expert witness is

whether the witness had any reasonable pretension to specialized knowledge on the

subject under investigation. If he does, he may testify and the weight to be given to such

testimony is for the trier of fact to determine. It is also well established that a witness

may be qualified to render an expert opinion based on training and experience. . . .

Zak v. Prudential Property & Casualty Insurance Company, 713 A.2d 681, 689 (Pa. Super. 1998),

(citing Erdos v. Bedford Valley Petroleum Co., 682 A.2d 806, 808 (Pa. Super. 1996), quoting Miller v.

Brass Rail Tavern, Inc., 664 A.2d 525, 528 (Pa. 1995)).

“Traditionally, for medical evidence to be deemed competent, a litigant’s expert witness need

only render an opinion within a reasonable degree of medical certainty.” Barbour v. Pa. Dep’t of

Transp., 732 A.2d 1157, 1160 (Pa. 1999) (citing Mitzelfelt v. Kamrin, 584 A.2d 888 (Pa. 1990)). If an

expert states an opinion, the expert must state the facts or data on which the opinion is based. Pa.R.E.

705.

As applied here, the facts indicate that Rachel is a board certified forensic pathologist with

twenty years of forensic pathology experience and that she possesses the requisite training to rule upon

the effects of poisons, including arsenic, on the human body. In order for the Commonwealth to prove

that arsenic was the cause of Brian’s death, it will need to show that Brian had arsenic poisoning in his

system and that this poisoning resulted in his death. The Commonwealth would have to show that

Rachel’s specialized knowledge in the area of forensic pathology is beyond that possessed by the

average layperson. This should not be a problem as most lay people do not have specialized training in

the area of poisoning and its effects on the human body. The Commonwealth will also be able to show

that Rachel’s specialized knowledge will be able to assist the jury in understanding the evidence,

1 “[T]he Comment to Rule 702 makes clear that this rule reflects our Commonwealth’s adoption of the Frye standard which

allies the ‘general acceptance’ test for admissibility.” Commonwealth v. Walker, 92 A.3d 766, 780 (Pa. 2014) citing

Pa.R.E.702 cmt.

Page 26: OCTOBER 2020 PENNSYLVANIA BAR EXAMINATIONQuestion No. 1 Susan and Doug are a married couple who reside in Steel Town, Pennsylvania. They have been married for 25 years and have one

including the effects that arsenic poisoning has on a person. Further, the facts expressly state that all of

Rachel’s work was performed using generally accepted methods in the field of forensic pathology which

should meet the requirement of Pa.R.E. 702(c). Since the cause of death, namely arsenic poisoning, will

be a key issue in the trial, and Rachel has the training, twenty years of experience in forensic pathology

and board certification, to assist the jury in reaching a determination on this issue, and Rachel’s

methodology is generally accepted in her field, the Commonwealth should have no difficulty qualifying

her as an expert witness to testify in these areas.

When Rachel testifies and renders an opinion with regard to the impact that arsenic poisoning

has on the human body, she will need to do so within a reasonable degree of medical certainty in the

field of forensic pathology which is the area for which she is being offered as an expert. She will also

have to outline the facts upon which her opinion is based. As long as Rachel testifies that she has

reached her conclusions as to the cause and manner of death within a reasonable degree of certainty in

those fields, and she outlines the facts which form the basis of her opinion, Rachel’s testimony should be

admissible.

3. If Louise testifies at Mary Beth’s homicide trial, the charge of robbery would be admissible

against her for impeachment purposes, the charge of DUI would not be admissible and the

charge of burglary would be admissible if the release from confinement, if any, was less

than 10 years and if greater than 10 years it would be admissible only if proper notice of its

intended use was given by the defense to the Commonwealth and the court finds that its

probative value outweighs its prejudicial effect.

“For the purpose of attacking the credibility of any witness, evidence that the witness has been

convicted of a crime, whether by verdict or by plea of guilty or nolo contendere, must be admitted if it

involved dishonesty or false statement.” Pa.R.E. 609(a). Although convictions can be used to attack the

credibility of any witness, there are limits on using such evidence after ten years. Pa.R.E. 609(b) sets

forth the following limitations:

(b) Limit on Using the Evidence After 10 Years. This subdivision (b) applies if more

than 10 years have passed since the witness’s conviction or release from confinement for

it, whichever is later. Evidence of the conviction is admissible only if:

(1) its probative value substantially outweighs its prejudicial effect; and

(2) the proponent gives an adverse party reasonable written notice of the intent to

use it so that the party has a fair opportunity to contest its use.

The crimes of robbery and burglary are crimes involving dishonesty for which a defendant can

be impeached. Commonwealth v. McEnany, 732 A.2 1263, 1271 (Pa. Super. 1999) citing

Commonwealth v. Strong, 563 A.2d 479 (Pa. 1989). A prior conviction for DUI is not in the nature of

crimen falsi and evidence of defendant’s conviction for the same is not admissible under Rule 609.

Commonwealth v. Hyland, 875 A.2d 1175, 1187 (Pa. Super. 2005), (citing Commonwealth v. Vitale, 664

A.2d 999, 1000 (Pa. Super 1995)).

As applied here, Louise’s robbery conviction would be admissible because it is a crime involving

dishonesty or false statement and it occurred within ten years of the date of trial. The DUI conviction

does not involve a crime of dishonesty or false statement and would therefore not be admissible. The

Page 27: OCTOBER 2020 PENNSYLVANIA BAR EXAMINATIONQuestion No. 1 Susan and Doug are a married couple who reside in Steel Town, Pennsylvania. They have been married for 25 years and have one

crime of burglary has been determined to be a crimen falsi offense but the conviction occurred fifteen

years ago. If Louise was confined for the burglary conviction and her release occurred less than 10

years from the date of the current trial, the conviction would likely be admissible. If Louise was not

confined for the burglary conviction, or was confined but was released more than 10 year before the date

of the current trial, the court would have to weigh whether the probative value substantially outweighs

its prejudicial effect. Further, the defense would have to show that it gave the Commonwealth

reasonable written notice of intent to use the burglary conviction so that the Commonwealth had a fair

opportunity to contest its use. If the defense properly gave notice of its intention to use the burglary

conviction for impeachment purposes, the conviction will be admissible if the court finds that the

probative value outweighs its prejudicial effect. If the court does not find that the probative value

outweighs the prejudicial effect, the conviction for burglary will likely not be admissible.

4. If Louise wants to file a divorce action against Frank in Pennsylvania, the Pennsylvania

courts will have jurisdiction over the action and the action can be filed in C County, PA.

23 Pa. C.S.A. § 3104(a) provides in pertinent part that “[t]he courts shall have original

jurisdiction in cases of divorce . . . ” That statute also provides,“[n]o spouse is entitled to commence an

action for divorce or annulment under this part unless at least one of the parties has been a bona fide

resident in this Commonwealth for at least six months immediately previous to the commencement of

the action. Both parties shall be competent witnesses to prove their respective residence, and proof of

actual residence within this Commonwealth for six months shall create a presumption of domicile within

this Commonwealth.” 23 Pa. C.S.A. §3104(b). The Pennsylvania Superior Court stated,“Our Divorce

Code requires a six-month residency period in order to maintain an action for divorce, and domiciliary

intent is inferred from the residence.” Gaboury v. Gaboury, 988 A.2d 672, 675 (Pa. Super. 2009).

As applied here, the facts indicate that Louise relocated to C County, PA on November 1, 2019,

with the intent to permanently reside in C County. Frank remained in Ohio with the intent to

permanently reside there. Louise has been a resident of Pennsylvania for over six months and has

expressed her intention to permanently reside there. Accordingly, there is a presumption that Louise is

domiciled in Pennsylvania. Thus, the Pennsylvania Courts will have jurisdiction over their divorce

action.

With regard to venue, Pa.R.C.P. No. 1920.2 provides in pertinent part as follows:

(a) The action, except a claim for custody, may be brought only in the county

(1) in which the plaintiff or the defendant resides, or

(2) upon which the parties have agreed

(i) in a writing which shall be attached to the complaint, or

(ii) by participating in the proceeding.

The facts indicate that Louise relocated to C County on November 1, 2019, where she intended

to permanently reside. Frank remained in Ohio where he intended to permanently reside. Since Frank

resides outside the Commonwealth, venue would be proper in C County where Louise resides under

Page 28: OCTOBER 2020 PENNSYLVANIA BAR EXAMINATIONQuestion No. 1 Susan and Doug are a married couple who reside in Steel Town, Pennsylvania. They have been married for 25 years and have one

Pa.R.C.P. No. 1902.2(a)(1) above. While Pa.R.C.P. No. 1902.2(a)(2) provides the proceeding may be

brought in a venue upon which the parties have agreed in writing, there is nothing in the facts to indicate

that the parties agreed on a venue. Therefore, as Frank resides in Ohio and Louise resides in C County,

C County would be the appropriate venue.

Page 29: OCTOBER 2020 PENNSYLVANIA BAR EXAMINATIONQuestion No. 1 Susan and Doug are a married couple who reside in Steel Town, Pennsylvania. They have been married for 25 years and have one

Question No. 3: Grading Guidelines

1. Criminal Law.

Comments: Candidates should identify first degree murder as the most serious homicide charge, discuss

the applicable facts which support that charge and conclude the charge is likely supported by the facts.

6 Points

2. Evidence.

Comments: Candidates should identify the evidentiary rules regarding the qualification and

admissibility of expert witnesses, including the standard to be met by experts when rendering expert

opinions, and apply the relevant facts.

5 Points

3. Evidence.

Comments: Candidates should identify the evidentiary rules governing impeachment of witnesses and

conclude that the robbery conviction will likely be admissible, the DUI conviction will not be

admissible, and the admissibility of the burglary conviction will depend on how the Court rules on

whether the probative value of the conviction substantially outweighs its prejudicial effect.

5 Points

4. Family Law.

Comments: Candidates should identify the rules governing jurisdiction and venue in divorce actions and

conclude that jurisdiction has properly been established in Pennsylvania and the action should be

brought in C County, Pennsylvania.

4 Points

Page 30: OCTOBER 2020 PENNSYLVANIA BAR EXAMINATIONQuestion No. 1 Susan and Doug are a married couple who reside in Steel Town, Pennsylvania. They have been married for 25 years and have one

Question No. 4

Jon and Ann Cohen moved to Smalltown in State A last year when Jon started teaching in

Smalltown School District at Smalltown High School, the public high school. Jon and Ann were

raised Jewish; belong to a Jewish synagogue where they regularly attend services and observe

Jewish traditions, customs, and holidays. Their 15-year-old son, Dave, who is also Jewish and is

a sophomore at Smalltown High School, plays trumpet in the Smalltown High School marching,

concert, and jazz bands; and he sings tenor in the Smalltown High School chorus. The

Smalltown High School concert band and chorus perform concerts every winter and spring. The

first year Dave attended Smalltown High School, the chorus performed an Easter program called

“ARISE.” ARISE consists of 12 original songs based on Christian teachings about the life, death,

and resurrection of Jesus Christ. Some songs include words of worship and “invitations” to

follow the teachings of Jesus. During the concert, a slideshow behind the chorus showed images

of a cross, an empty tomb, Jesus performing miracles, and individuals kneeling in prayer.

Christian scriptures were recited between musical pieces, and a printed program provided phone

numbers for local Christian clergy inviting concert attendees to call: “If you would like to know

more about the Easter Story.”

Jon, Ann, and Dave were all bothered that Dave’s entire spring semester of music

education was devoted to the program and that Dave could not receive music education at

Smalltown High School without participating in the Easter program. The Smalltown School

District superintendent told Jon that no one else had complained over the years about the

program, and that it would continue indefinitely.

Ann got a job in the records department at the Smalltown office of RuralMedGroup

(RMG), a State A corporation with 20 offices and 300 employees in State A. RMG hired Ann

because new federal regulations strictly limit deadlines to submit reimbursement requests for

government-funded healthcare. On her first day, Ann learned that she was expected to work 4

26

Page 31: OCTOBER 2020 PENNSYLVANIA BAR EXAMINATIONQuestion No. 1 Susan and Doug are a married couple who reside in Steel Town, Pennsylvania. They have been married for 25 years and have one

hours every Saturday to input data from the prior week. When Ann informed RMG that she

absolutely does not work from sundown on Friday to sundown on Saturday in observance of the

Jewish Sabbath, her supervisor explained that the data had to be inputted between 5:00 p.m.

Friday (the close of business) and 5:00 p.m. Saturday. Otherwise, RMG would lose hundreds of

thousands of dollars in revenue and risk fines. The data input must be done on-site at each

office, and RMG does not have a qualified employee within a 100-mile radius that could cover the

Saturday shift. It would be extremely expensive for RMG to hire a second employee to perform

that work. Accordingly, RMG terminated Ann’s employment.

Jon and Ann met with attorney Jessica Dennis to discuss both the Easter program and

Ann’s termination by RMG. Dennis explained that as Dave’s parents and on his behalf they

could challenge the Smalltown High School practice of performing an Easter program every year

under the United States Constitution. Dennis further told Anne that she may be able to bring

claims against RMG under Title VII of the Civil Rights Act of 1964 (Title VII) and State A’s

Religious Discrimination Act.

1. Assuming for this question only that all jurisdictional and procedural requirements

are satisfied, what doctrine under the United States Constitution should the

Cohens raise on Dave’s behalf to challenge the Smalltown School District practice of

performing an Easter program every year similar to the ARISE program, and with

what likely result?

2. After exhausting her administrative remedies, Ann properly filed a lawsuit in the

appropriate state court in State A against RMG alleging that RMG’s termination of

her employment violated both Title VII and State A’s Religious Discrimination Act.

If RMG determines it would prefer to litigate both of Ann’s claims in federal district

court rather than state court, what action should RMG take and, assuming RMG

acts within the required time limits, with what likelihood of success will RMG have

regarding having both of Ann’s claims heard in federal district court?

3. Assume for this question that RMG has stipulated that Ann can establish a prima

facie case under Title VII for failure to accommodate a religious practice. Other

than rebutting Ann’s prima facie case, what defense should RMG raise to Ann’s

Title VII claim, and what is the likely outcome?

27

Page 32: OCTOBER 2020 PENNSYLVANIA BAR EXAMINATIONQuestion No. 1 Susan and Doug are a married couple who reside in Steel Town, Pennsylvania. They have been married for 25 years and have one

Question No. 4: Examiner’s Analysis

1. The Cohens Should Bring a First Amendment Establishment Clause Challenge and

Will Likely Be Successful.

The “Establishment Clause” of the First Amendment of the United States Constitution

provides that “Congress shall make no law respecting an establishment of religion.” U.S. Const.

amend. I (emphasis added). The First Amendment is applicable to the states and local

governments through the Fourteenth Amendment. Everson v. Bd. of Educ., 330 U.S. 1, 14-18

(1947).

The Supreme Court has identified “six rough categories” of Establishment Clause cases

that have been decided since it was held to be applicable to the states. Am. Legion v. Am.

Humanist Ass’n, __U.S.__, 139 S. Ct. 2067, 2081 n.16 (2019) (plurality opinion). Relevant here

is the line of cases involving “religious expression in public schools.” Id. (citing School Dist. of

Abington Twp. v. Schempp, 374 U.S. 203 (1963) and Lee v. Weisman, 505 U.S. 577 (1992)).

“[T]here are heightened concerns with protecting freedom of conscience from subtle coercive

pressure in the elementary and secondary public schools.” Lee, 505 U.S. at 592. In Lee, the

Supreme Court held that a non-sectarian prayer during a public school graduation violated the

Establishment Clause. Id. at 599.

There are several ways that the trial court here might analyze the Cohens’ constitutional

challenge to the Easter program. A starting point would likely be the test promulgated in Lemon

v. Kurtzman, 403 U.S. 602 (1971).1 In Lemon, the Supreme Court enumerated a three-part test

that has come to be known as the “Lemon test,” for whether state action violates the

Establishment Clause of the First Amendment. Id. at 612-13 (citations omitted). “Under the

Lemon Test, a court must ask whether a challenged government action (1) has a secular purpose;

(2) has a ‘principal or primary effect’ that ‘neither advances nor inhibits religion’; and (3) does

not foster ‘an excessive government entanglement with religion.’” Am. Legion, 139 S. Ct. at

2078-79 (citing Lemon, 403 U.S. at 612-13).

Additionally, or alternatively, the trial court could analyze the Cohen’s challenge of the

Easter program under the “endorsement test” articulated by Justice O’Connor in Lynch v.

Donnelly, 465 U.S. 668 (1984) (O’Connor concurring).2 The endorsement test arose out of the

1 While the Lemon test has been criticized and inconsistently applied, it has not been overruled in the context of

challenges to religious practices in public schools. The Supreme Court recently revisited the Lemon test in Am.

Legion v. Am. Humanist Ass’n, __U.S.__, 139 S. Ct. 2067 (2019), with the test being described as “harshly criticized

by Members of this Court, lamented by lower court judges, and questioned by a diverse roster of scholars.” Id. at

2081 (plurality opinion) (notes and citations omitted). With that said, and to the disappointment of at least one

Justice, the Court did not overrule the Lemon test, but rather, in American Legion, declined to apply it “in cases,

including the one now before us, that involve the use, for ceremonial, celebratory, or commemorative purposes, of

words or symbols with religious associations.” Id. at 2081; see also, Id. at 2097 (“I would take the logical next step

and overrule the Lemon test in all contexts.”) (Thomas, J., concurring).

2 “[The] ‘endorsement test’ is now widely accepted as the controlling analytical framework for evaluating

Establishment Clause claims. It would be wrong, however, to suggest the [Supreme] Court is unanimous in its

adoption of the endorsement test.” Bauchman v. West High Sch., 132 F.3d 542, 552 (10th Cir. 1997) (citation

omitted).

28

Page 33: OCTOBER 2020 PENNSYLVANIA BAR EXAMINATIONQuestion No. 1 Susan and Doug are a married couple who reside in Steel Town, Pennsylvania. They have been married for 25 years and have one

following interpretation of Lemon: “[t]he purpose prong of the Lemon test asks whether

government’s actual purpose is to endorse or disapprove of religion. The effect prong asks

whether, irrespective of government’s actual purpose, the practice . . . in fact conveys a message

of endorsement or disapproval.” Id. at 690. The endorsement test would look at both “the

subjective and objective components of the message communicated by government action,” and

would hold action that demonstrates such endorsement or disapproval violates the First

Amendment. Id. See also County of Allegheny v. ACLU, 492 U.S. 573, 592-93 (1989) (“In

recent years, we have paid particularly close attention to whether the challenged governmental

practice either has the purpose or effect of ‘endorsing’ religion, a concern that has long had a

place in our Establishment Clause jurisprudence.) (citations omitted). Finally, it is possible that

the trial court here might simply adopt reasoning from cases that predated or declined to apply

either the Lemon or the endorsement test.

Applying both the Lemon test and the endorsement test, federal appellate courts have held

that some integration of religious music into public school music programs is permissible under

the Establishment Clause. Bauchman v. West High Sch., 132 F.3d 542 (10th Cir. 1997); Doe v.

Duncanville Indep. Sch. Dist., 70 F.3d 402, 408 (5th Cir. 1995). These decisions turned largely

on the fact that the programs included an array of secular and non-secular music and/or the

myriad of secular reasons that a public school music program would include religious music.

Bauchman, 132 F.3d at 554 (“[I]t is recognized that a significant percentage of serious choral

music is based on religious themes or text. Any choral curriculum designed to expose students

to the full array of vocal music culture therefore can be expected to reflect a significant number

of religious songs.”); Doe, 70 F.3d at 407 (noting that the challenged religious music was

“particularly useful to teach students to sight read and sing a capella” and was composed by a

reputable composer).

In contrast, there are also cases in which federal courts have found violations of the

Establishment Clause in connection with religious music and music programs in public schools,

primarily because the music was seen as more akin to prayer or the program incorporated

religious elements beyond the music itself. See, e.g., Freedom from Religion Found. v. Concord

Cmty. Sch., 148 F.Supp.3d 727, 741 (N.D. Ind. 2015) (holding that inclusion of a living nativity

scene at a public school program “conveys a message of endorsement of religion, or that a

particular religious belief is favored or preferred,” and, therefore violates the Establishment

Clause); S.D. v. St. Johns Cty. Sch. Dist., 632 F.Supp.2d 1085, 1095 (M.D. Fla. 2009) (holding

that requiring third grade students to practice, learn and perform a song called “In God We Still

Trust” violates the Establishment Clause); Doe v. Aldine Indep. Sch. Dist., 563 F. Supp. 883, 888

(S.D. Tex. 1982) (holding that singing a “school prayer” at school events violates the

Establishment Clause, premised on the fact that it is “prayer”).

Here, whether a court applied the Lemon test, the endorsement test, or some combination

of the two, it would likely hold that the music program at issue violates the Establishment clause.

Of course, it could be argued that the program has the secular purpose of teaching music to

students. However, it is unclear what secular purpose the inclusion of scripture readings,

religious images and referrals to clergy might advance. Similarly, under the second prong of

Lemon, it is difficult to see how these elements of the program would have a “primary effect”

29

Page 34: OCTOBER 2020 PENNSYLVANIA BAR EXAMINATIONQuestion No. 1 Susan and Doug are a married couple who reside in Steel Town, Pennsylvania. They have been married for 25 years and have one

other than to advance religion. Finally, the amount of time that music students prepare for the

program and the fact that a student cannot receive music education unless he participates

evidences “‘an excessive government entanglement with religion,” as prohibited by Lemon.

The endorsement test, of course, is derived from the Lemon test, and would likely compel

the same conclusion under that test. The fact that every piece in the program is religious and

original (as opposed to sacred pieces of classical music), coupled with the inclusion of Scripture

readings, religious images and proselyting messages, would likely compel the court to conclude

that both the purpose of the program is to endorse religion and the actual effect conveys a

message of endorsement. Lynch, 465 U.S. at 690.

If the Cohens were challenging the music selections alone, there may be some precedent

on which the Smalltown School District (STSD) could base a defense. However, the Easter

programs at Smalltown High School (SHS) are not akin to music programs that incorporates

religious music into a repertoire of religious and non-religious pieces. All of the music is

religious, and, more significantly, includes both the performance of religious music and the

inclusion of religious symbols, scripture readings, and the invitation to spiritual counsel from

clergy. This sets it apart from cases like Bauchman and Doe.

Finally, even if a court analyzes the challenge to the Easter program in light of public

school cases that pre-date Lemon or were decided on other grounds, the program is likely to be

held unconstitutional. In Engle v. Vitale, 370 U.S. 421 (1962), the Supreme Court held that “by

using its public school system to encourage recitation of [a brief non-sectarian prayer], the State

of New York has adopted a practice wholly inconsistent with the Establishment Clause.” Id. at

424. In Abington Sch. Dist. v. Schempp, 374 U.S. 203 (1963), the Supreme Court held that

requiring Bible reading and/or the recitation of the Lord’s Prayer in public schools violates the

First Amendment. Id. at 223.3 While both Engle and Schempp pre-date Lemon, the Court’s

“most prominent modern case” on the issue of religious practices in schools, Lee v. Weisman,

505 U.S. 577 (1992), declined to reconsider Lemon in that context. Id. at 587. In Lee, the Court

held that non-sectarian prayers during a public school graduation ceremony violates the

Establishment Clause. Id. at 587, 599. The Court noted that attendance and participation at

graduation are “in a fair and real sense obligatory, [even] though the school district does not

require attendance as a condition for receipt of the diploma.” Id. at 586. The Court’s primary

concern was the risk of coercion and the extensive involvement of the school, noting the

extensive involvement with a school official deciding there would be a prayer, selecting the

clergy person to deliver the prayer, and providing guidelines to the clergy on the content of the

prayer. Id. at 587-89, 590 (“The degree of school involvement here made it clear that the

graduation prayers bore the imprint of the State and thus put school-age children who objected in

an untenable position.”); see also Id. at 592 (“As we have observed before, there are heightened

concerns with protecting freedom of conscience from subtle coercive pressure in the elementary

and secondary public schools.”) (citations omitted).

3 In so holding, the court noted that the exercises were part of curricular activities during the compulsory school day

and they were performed in school buildings under the supervision of and with participation of teachers. Abington

Twp., 374 U.S. at 223.

30

Page 35: OCTOBER 2020 PENNSYLVANIA BAR EXAMINATIONQuestion No. 1 Susan and Doug are a married couple who reside in Steel Town, Pennsylvania. They have been married for 25 years and have one

Here, of course, the music program is sectarian (unlike Engle) and incorporates Bible

reading (like Abington). Moreover, while it appears that participation in the music program itself

is not compulsory, for any student that wants to participate in the music program, they have to

participate in the ARISE program. Thus, it is likely that the same concerns present in Lee about

the risk of coercion and the extensive involvement of school personnel with religious matters are

present, further compelling the conclusion that the program violates the Establishment Clause.

For the reasons set forth above, it is likely that the Cohens’ Establishment Clause

challenge will be successful.

2. RuralMedGroup (RMG) Should File a Notice of Removal of Ann’s Lawsuit to

Federal Court Based on Federal Question Jurisdiction and Supplemental

Jurisdiction Over her State Law Claim.

a. RMG Will Be Able to Remove Ann’s Lawsuit to Federal Court Because Her

Title VII Claim Arises Under Federal Law (Federal Question Jurisdiction)

28 U.S.C.S. §1441 provides, in relevant part:

[A]ny civil action brought in a State court of which the district courts of the United States

have original jurisdiction, may be removed by the defendant or the defendants to the

district court of the United States for the district and division embracing the place where

such action is pending.

Thus, the question of whether RMG can remove Ann’s claims to federal court turns upon

whether her action is one “of which the district courts of the United States have original

jurisdiction.” Id. The United States Supreme Court stated, “This Court has long held that a

district court, when determining whether it has original jurisdiction over a civil action, should

evaluate whether that action could have been brought originally in federal court.” Home Depot

U.S.A., Inc. v. Jackson, 587 U.S.__, 139 S.Ct. 1743, 1748 (2019) (citations omitted). Original

jurisdiction is proper in federal court where the plaintiff’s complaint could have been filed

“either because it raises claims arising under federal law or because it falls within the court’s

diversity jurisdiction.” Id. (citations omitted). Generally, federal courts have jurisdiction over

two types of cases – those “arising under” federal law, and cases where there is diversity of

citizenship among the parties and the amount in controversy exceeds $75,000. 28 U.S.C. §§

1331, 1332(a). These are usually referred to, respectively, as federal question jurisdiction and

diversity jurisdiction. Jackson, 587 U.S. at __, 139 S.Ct. at 1746.

Here, RMG is a State A corporation and its offices are also located in State A, and Ann

lives in State A. Therefore, Ann’s action would not have fall within the federal’s court’s

diversity jurisdiction. See 28 U.S.C. § 1332. However, Ann’s complaint does state a claim for

employment discrimination under Title VII, a federal statute. 42 U.S.C. § 2000e-2. Notably, not

only is Title VII a law promulgated by Congress (i.e., a federal law), but the statute itself

includes a provision conferring jurisdiction for Title VII claims in United States district courts.

28 U.S.C. § 2000e-5(f)(3). Because Ann’s Title VII claim is one “arising under” federal law,

and Title VII specifically confers jurisdiction in U.S. district courts, RMG will be able to remove

31

Page 36: OCTOBER 2020 PENNSYLVANIA BAR EXAMINATIONQuestion No. 1 Susan and Doug are a married couple who reside in Steel Town, Pennsylvania. They have been married for 25 years and have one

Ann’s Title VII case to federal court. Notably, since subject matter jurisdiction is conferred

based on a federal question and not diversity of citizenship, the prohibition on removal by a

defendant who is a citizen of the State in which the action is brought is inapplicable here. 28

U.S.C. § 1441(b)(2).

b. The Federal Court Can Exercise Supplemental Jurisdiction Over Ann’s State

Law Claim.

Where diversity of citizenship is not present, a federal court has discretion to exercise

supplemental or pendant jurisdiction over state law claims. 28 U.S.C. §1367(a) provides that,

subject to certain limitations, “in any civil action of which the district courts have original

jurisdiction, the district courts shall have supplemental jurisdiction over all other claims that are

so related to claims in the action within such original jurisdiction that they form part of the same

case or controversy under Article III of the United States Constitution.” See also Exxon Mobile

Corp. v. Allapattah Servs., Inc., 545 U.S. 546, 552 (2005).

Here, Ann’s federal Title VII religious discrimination claim and her state RDA claim

arise out of the exact same facts. Specifically, Ann has sued under both Title VII and the RDA

based on RMG’s failure to accommodate her religious practice. Accordingly, it is likely that the

federal district court would exercise supplemental jurisdiction over Ann’s RDA claim and hear

the matter in one proceeding.4

3. RMG Should Raise the Defense of Undue Hardship and Will likely Be Successful.

Title VII of the Civil Rights Act of VII prohibits employers from discriminating in the

terms and conditions of an individual’s employment “because of such individual’s . . . religion.”

42 U.S.C. § 2000e-2(a)(1). “[Under Title VII], [t]he word ‘religion’ is defined to ‘includ[e] all

aspects of religious observance and practice, as well as belief, unless an employer demonstrates

that he is unable to reasonably accommodate to’ a ‘religious observance or practice without

undue hardship on the conduct of the employer’s business.’ ” E.E.O.C. v. Abercrombie & Fitch

Stores, Inc., 575 U.S. 768, __, 135 S.Ct. 2028, 2032 (2015) (citing and quoting 42 U.S.C. §

2000e(j)); see also, Tabura v. Kellogg USA, 880 F.3d 544, 549 (10th Cir. 2018) (“Title VII, thus,

requires that ‘an employer, short of “undue hardship,” make “reasonable accommodations” to the

religious needs of its employees.’”) (quoting Trans World Airlines, Inc. v. Hardison, 432 U.S.

63, 66 (1977)).

Assuming that an employee can establish a violation of Title VII based on an employer’s

failure to accommodate the employee’s religion, the employer must then show either that it

provided a reasonable accommodation5 or that doing so would cause an undue hardship. Tabura,

880 F.3d at 550 (citing Thomas v. Nat’l Ass’n of Letter Carriers, 225 F.3d 1149, 1156 (10th Cir.

4 While 28 U.S.C. §1367(c) provides for several circumstances in which a court may decline to exercise

supplemental jurisdiction, there is no indication in the facts indicating that any of these factors are applicable here.

5 The determination of whether an accommodation is “reasonable” is fact-intensive and must be decided on a case-

by-case basis, and usually by the fact finder. Tabura, 880 F.3d at 551, 555 (citations omitted).

32

Page 37: OCTOBER 2020 PENNSYLVANIA BAR EXAMINATIONQuestion No. 1 Susan and Doug are a married couple who reside in Steel Town, Pennsylvania. They have been married for 25 years and have one

2000). Here, it is undisputed that RMG did not make any effort to accommodate Ann’s religious

practice of observing the Sabbath. Thus, unless RMG can show that it would suffer undue

hardship in making an accommodation, Ann will succeed in her claim.

RMG can show that it would suffer undue hardship if it would “‘bear more than a de

minimis cost in order to give an employee Saturdays off to observe [the] Sabbath.’” Tabura, 880

F.3d at 557 (quoting TWA, 432 U.S. at 84). “Any cost in efficiency or wage expenditure that is

more than de minimis constitutes undue hardship. The cost of hiring an additional worker or

the loss of production that results from not replacing a worker who is unavailable due to a

religious conflict can amount to undue hardship.” Lee v. ABF Freight Sys., Inc., 22 F.3d 1019,

1023 (10th Cir. 1994) (emphasis added) (internal quotations and citations omitted). Like the

question of the reasonableness of an accommodation, the issue of undue hardship “is a fact

question that turns on the particular context of each case.” Tabura, 880 F.3d at 558 (internal

quotations and citations omitted).

Here, Ann is the only records staff person within 100 miles of RMG’s office. Further,

not having a records person work on Saturdays will result in significant cost to RMG. It is likely

that such a requirement would cause more than “de minimis cost” to RMG. Accordingly, RMG

is likely to be able to prove that providing an accommodation for Ann’s Sabbath observance

would likely cause RMG an undue hardship.

33

Page 38: OCTOBER 2020 PENNSYLVANIA BAR EXAMINATIONQuestion No. 1 Susan and Doug are a married couple who reside in Steel Town, Pennsylvania. They have been married for 25 years and have one

Question No. 4: Grading Guidelines

1. First Amendment – Establishment Clause

Comments: Candidates should be able to identify that a set of facts gives rise to a challenge

under the Establishment Clause of the First Amendment, apply the applicable legal standard to

those facts, and come to a well-reasoned conclusion.

8 points.

2. Removal, Federal Question Jurisdiction and Supplemental Jurisdiction

Comments: Candidates should identify removal as the proper procedural action to transfer a case

from state court to federal court and analyze the bases for both federal question and supplemental

jurisdiction as applied to a specific set of facts to reach a well-reasoned conclusion.

7 points.

3. Undue Hardship Defense to Title VII Religious Accommodation Claim

Comments: Candidates should identify undue hardship as an applicable defense to a Title VII

religious discrimination claim based on failure to accommodate, and apply the appropriate legal

standard to a set of facts to arrive at a well-reasoned conclusion.

5 points.

34

Page 39: OCTOBER 2020 PENNSYLVANIA BAR EXAMINATIONQuestion No. 1 Susan and Doug are a married couple who reside in Steel Town, Pennsylvania. They have been married for 25 years and have one

Question No. 5

Charles, an eccentric media tycoon, owned Blackacre, a large tract of vacant land in Big

City, Pennsylvania. In June 2015, Charles conveyed Blackacre pursuant to a valid deed that

stated, “To Big City in fee simple upon the condition that Blackacre be used as a park. If

Blackacre is ever not used as a park, the grantor or his heirs may re-enter the property and

terminate the estate.” Charles died in December 2019. Charles’s will left his entire estate,

including any interest in real estate that he owned at his death, to his daughter, Sarah.

After settling Charles’s estate, Sarah decided to hold a public sale of the trove of books,

coins, and other personal property that Charles had collected over his lifetime. Sarah hired Walt

to provide various services related to the sale including advertising the sale throughout Big City,

posting pictures of the items to be sold with their prices on various social media outlets, and

serving as Sarah’s agent to sell the items. Walt told Sarah, “I’m going to contract with Rosebud,

a Big City company, to print yard signs and flyers advertising the sale and I will place and

distribute the signs and flyers throughout Big City.” Sarah told Walt, “That’s fine with me. I

like Rosebud’s work. If I were doing this myself, I would have contracted with Rosebud.” In

their valid written contract, Sarah agreed to pay Walt a flat fee for his services and to “bear any

printing costs incurred by Walt.” Rosebud charged Walt $5,000 for the printing.

One of the items included for public sale was a rare 1943 Copper Penny. The price of the

coin was listed as $100,000 on social media, in the flyers, and in Walt’s handwritten notebook of

prices of items for sale.

Jed and his wife, Mary, lived at Xanadu, a mansion in Big City that they owned as tenants

by the entireties. Jed, an avid coin collector, saw the coin was for sale for $100,000 on social

media and told Mary, “I’m going to buy that coin for my collection.” Mary replied, “I’m sick and

tired of you spending all of our money on your dumb hobby. If you spend any more money

buying coins, I’ll divorce you.” Without Mary’s consent or knowledge, Jed signed a quitclaim

35

Page 40: OCTOBER 2020 PENNSYLVANIA BAR EXAMINATIONQuestion No. 1 Susan and Doug are a married couple who reside in Steel Town, Pennsylvania. They have been married for 25 years and have one

deed conveying any interest that Jed had in Xanadu for $100,000 to “Foster-Kane & Co.,” a

company that speculated in risky real estate ventures, to obtain the money to buy the coin.

On the day of sale, Walt misread the handwriting in his notebook and erroneously marked

the coin for sale at $10,000 instead of $100,000. Aware of Walt’s error in marking the sale price

of the coin, Jed immediately signed a simple agreement to buy the coin from Sarah for $10,000.

The agreement did not mention who bore the risk of any mistake arising from the sale of the

coin.

1(a). What legal interests were created by the conveyance of Blackacre in 2015?

1(b). In August 2020, Big City announced that it had conveyed Blackacre to Nile, an e-

commerce company, and Nile started construction of its new corporate headquarters

on the property. Sarah heard about these actions but did nothing. Following the

actions of Big City and Nile, who is the current owner of Blackacre?

2. Dissatisfied with the poor results from the public sale, Sarah refused to pay Walt

for his services and for any costs that Walt incurred with Rosebud in producing the

yard signs and flyers. Due to a large number of other debts, Walt failed to pay, and

he is unlikely ever to be able to pay, Rosebud for printing the signs and flyers. What

theory of contract law would best support a suit by Rosebud against Sarah to collect the money

owed to it by Walt?

3. When Sarah learned that the 1943 Copper Penny was sold for the erroneously

marked price of $10,000 instead of for $100,000 as listed on social media, the flyers,

and in Walt’s notebook, Sarah sued Jed to rescind the sale based upon mistake.

What is the likelihood that Sarah’s suit based upon mistake will be successful?

4(a). Under Pennsylvania property law, what was the effect of Jed’s unilateral

conveyance of his interest in Xanadu to Foster-Kane & Co.?

4(b). Assume for purposes for this subpart only that Jed and Mary were divorced prior to

the time that Jed conveyed any interest that he had in Xanadu to Foster-Kane &

Co.. Applying Pennsylvania property law, who owns Xanadu following the divorce

and Jed’s unilateral conveyance of his interest in Xanadu to Foster-Kane & Co.?

36

Page 41: OCTOBER 2020 PENNSYLVANIA BAR EXAMINATIONQuestion No. 1 Susan and Doug are a married couple who reside in Steel Town, Pennsylvania. They have been married for 25 years and have one

Question No. 5: Examiner’s Analysis

1(a). The June 2015 deed conveying Blackacre created a fee simple subject to a condition

subsequent in Big City and a future interest known as a power of termination or

right of re-entry in Charles.

“A fee simple subject to a condition subsequent is a fee which is created in an instrument

of conveyance which provides that, upon the happening of some certain event, the grantor or his

successors in interest shall have the power to enter and terminate the estate of the grantee.”

RALPH E. BOYER ET AL., THE LAW OF PROPERTY: AN INTRODUCTORY SURVEY, § 6.2, at 128

(West Publishing Co. 4th ed. 1991); see also Emrick v. Bethlehem Twp., 485 A.2d 736, 739 (Pa.

1984). Generally, a deed is construed as creating a fee simple subject to a condition subsequent

if it satisfies two requirements. First, the deed must use “one of the following phrases . . . ‘upon

express condition that,’ or ‘upon condition that,’ or ‘provided that,’ or a phrase of like import.”

RESTATEMENT OF THE LAW OF PROPERTY, § 45, cmt. j (1936). Second, the deed must contain

“a provision that if the stated event occurs, the conveyor ‘may enter and terminate the estate

hereby conveyed,’ or a phrase of like import.” Id.

The June 2015 deed from Charles to Big City described in the facts satisfies the

Restatement requirements for creating a fee simple subject to a condition subsequent. The deed

uses the phrase “upon the condition.” The deed also states, “[i]f Blackacre is ever not used as a

park, the grantor or his heirs may re-enter the property and terminate the estate.” Based upon the

language in the deed, Charles granted Blackacre to Big City as a fee simple subject to a condition

subsequent.

Because the grantee’s interest can end upon the happening of a specified event, the

grantor is deemed to have retained a future interest in the property. BOYER, supra § 7.4, at p.

164/ see also Higbee Corp. v. Kennedy, 428 A.2d 592, 595 (Pa. Super. 1981). “The interest held

by the grantor in such cases has been termed a right of re-entry. This interest has also been called

a power of termination.” Emrick 485 A.2d at 739 (citations omitted). Because he conveyed

Blackacre to Big City as a fee simple subject to a condition subsequent, Charles has a future

interest - a power of termination/right of re-entry.

1(b). Although the condition in the 2015 deed occurred, Nile remains as the owner of

Blackacre because Sarah, who inherited the power of termination from Charles, has

not exercised her power to terminate Nile’s estate.

Pennsylvania law has long recognized that “[a] grantor and his successors are capable of

transmitting . . . a right of re-entry by inheritance, conveyance, assignment or release.” Higbee,

428 A.2d at 595 (citing London v. Kingsley, 81 A.2d 870, 873 (Pa. 1951)). The facts state that

Charles left his entire estate, including any interest in real estate that he owned at his death, to

Sarah. Therefore, through inheritance, Sarah received the power of termination concerning

Blackacre from Charles.1

1 The rule against perpetuities does not apply in this case for two reasons. First, the grantor’s right of re-entry/power

of termination is deemed vested at the time of its creation. Higbee, 428 A.2d at 595. Second, the facts state that

37

Page 42: OCTOBER 2020 PENNSYLVANIA BAR EXAMINATIONQuestion No. 1 Susan and Doug are a married couple who reside in Steel Town, Pennsylvania. They have been married for 25 years and have one

Unlike a fee simple determinable interest in land, a fee simple subject to a condition

subsequent does not automatically transfer title to the property to the grantor or his successors

upon the occurrence of the event specified in the deed. Instead, the grantor or his successors

must take some action to retake or perfect title upon the happening of the event stated in the

condition. Emrick, 485 A.2d at 739. Otherwise, the interest that has been created subject to the

condition subsequent “will continue until this power is exercised.” RESTATEMENT (FIRST) OF

PROPERTY, § 24.

An actual physical entry onto the property is not essential to exercise the right of

termination. Emrick, 485 A.2d at 740; RESTATEMENT (FIRST) OF PROPERTY, § 24, cmt. b,

Special Note. “The interest subject to such a power is terminated by any appropriate

manifestation, upon the part of the person in whose favor the condition exists, of his intent

thereby to terminate the interest in question.” Id.

The facts state that Big City conveyed Blackacre to Nile in August 2020. Thus, Nile

received the same title to Blackacre that Big City received from Charles: a fee simple subject to a

condition subsequent. The facts further state that Sarah heard about Big City’s conveyance of

Blackacre to Nile and Nile’s start of construction of its corporate headquarters on the property

contrary to the condition in the 2015 deed from Charles to Big City that Blackacre be used as a

park. The facts finally state that Sarah did nothing about these actions. Therefore, Nile is the

current owner of Blackacre in fee simple subject to a condition subsequent because Sarah, who

inherited the power of termination from Charles, has not exercised her power by some

appropriate manifestation of intent to terminate Nile’s estate.

2. Rosebud should be able to recover against Sarah under the contract theory that it is

a third party beneficiary of the contract between Sarah and Walt.

Since it is not a party to the contract, Rosebud would have to assert that it is a third party

beneficiary of the contract between Sarah and Walt to pursue a claim against Sarah. In Spires v.

Hanover Fire Ins. Co., 70 A.2d 828, 830 (Pa. 1950), the Pennsylvania Supreme Court held that,

in order for a third party beneficiary to have standing to recover on a contract, both contracting

parties must have expressed an intention that the third party be a beneficiary, and that intention

must have affirmatively appeared in the contract itself. Id.. “[I]n other words, a promisor cannot

be held liable to an alleged beneficiary of a contract unless the latter was within his

contemplation at the time the contract was entered into and such liability was intentionally

assumed by him in his undertaking.” Id. at 830-31.

In Guy v. Liederbach, 459 A.2d 744, 751 (Pa. 1983), the Pennsylvania Supreme Court

overruled Spires “to the extent that it states the exclusive test for third party beneficiaries.” In

doing so, the Supreme Court adopted Section 302 of the Restatement (Second) of Contracts as a

guide for analysis of third party beneficiary claims in Pennsylvania. Id. Under Section 302 of

the Restatement (Second) of Contracts, a third party can claim rights under a contract, even if

those rights are not explicitly stated, if it is consonant with the intention of the contracting

Charles’ conveyance of Blackacre during which he retained the power of termination/right of re-entry occurred in

June 2015. Pennsylvania, however, prospectively abolished the rule against perpetuities as of January 1, 2006. See

20 Pa. C.S.A. § 6107.1.

38

Page 43: OCTOBER 2020 PENNSYLVANIA BAR EXAMINATIONQuestion No. 1 Susan and Doug are a married couple who reside in Steel Town, Pennsylvania. They have been married for 25 years and have one

parties. As explained by the Court in Liederbach, the Restatement (Second) established a two-

part test for determining whether one is an intended third party beneficiary. The first part is a

standing requirement that leaves discretion with the court to determine whether recognition of

third party beneficiary status would be appropriate to effectuate the intent of the parties. The

second part defines the two types of claimants who can be intended beneficiaries. If a third party

satisfies both parts of the test, a claim may be asserted under the contract. Id. at 751.

In Scarpitti v. Weborg, 609 A.2d 147 (Pa. 1992), the Supreme Court revisited its decision

in Guy. The Scarpitti court declared that its decision in Liederbach established “a properly

restricted cause of action” for “a narrow class of third party beneficiaries . . . if they [could] show

that they meet the requirements of § 302.” Id. at 150. Thus, the Court in Scarpitti summarized

the law governing third party beneficiaries in Pennsylvania after Liederbach in this way:

[A] party becomes a third party beneficiary only where both parties to the contract

express an intention to benefit the third party in the contract itself, unless, the

circumstances are so compelling that recognition of the beneficiary’s right is

appropriate to effectuate the intention of the parties, and the performance satisfies

an obligation of the promisee to pay money to the beneficiary or the

circumstances indicate that the promisee intends to give the beneficiary the

benefit of the promised performance.

Scarpitti, 609 A.2d at 150-51 (emphasis in original) (citations omitted).

In this case, Sarah and Walt did not expressly state an intent in their contract to make

Rosebud a third party beneficiary. Nevertheless, Rosebud can rely upon the Restatement

(Second) test for an intended third party beneficiary to recover against Sarah. The facts state that

the underlying contract between Sarah, the promisor, and Walt, the promisee, required Sarah to

“bear any printing costs incurred by Walt.” In including language about the payment of the

printing cost incurred by Walt in their contract, the intention of the parties was to make Sarah the

party ultimately responsible for the costs incurred by Walt with a third party contractor. Thus,

recognizing Rosebud’s right as a third party to receive payment for its printing services directly

from Sarah would be “appropriate to effectuate the intention of the parties.” RESTATEMENT

(SECOND) OF CONTRACTS, § 302 (1) (Am. Law Inst. 1981). Further, the performance of the

promise would satisfy the obligation of promisee (Walt) to pay money to the beneficiary

(Rosebud).

Given these circumstances, Rosebud should be able to recover its $5,000 bill for its

printing work from Sarah under the theory that it is a third party beneficiary of the contract

between Sarah and Walt.

3. Even though the mistake about the price of the coin was unilateral in nature,

Sarah’s suit to rescind the sale of the 1943 Copper Penny to Jed nevertheless likely

will be successful because Jed knew that Walt had erroneously marked the price of

the coin at $10,000 instead of $100,000 and is not permitted to “snap up” the coin

for the lower price as a result of a unilateral mistake.

39

Page 44: OCTOBER 2020 PENNSYLVANIA BAR EXAMINATIONQuestion No. 1 Susan and Doug are a married couple who reside in Steel Town, Pennsylvania. They have been married for 25 years and have one

Pennsylvania case law holds that when a party who is adversely affected by a mistake in

a written contract seeks relief through the courts, the relief granted depends on the nature and

effect of that mistake. Lanci v. Metropolitan Ins. Co., 564 A.2d 972, 974 (Pa. Super. 1989).

When both parties to a contract are mistaken about a fact relating to the contract, the mistake is

mutual. For relief to be granted based on a mutual mistake, “[t]he mistake must relate to the

basis of the bargain; it must materially affect the parties’ performance; and, it must not be one as

to which the injured party bears the risk.” Loyal Christian Ben. Ass’n v. Bender, 493 A.2d 760,

762 (Pa. Super. 1985).

When only one of the parties has an erroneous belief at the time of formation about a fact

relating to the contract, the mistake is unilateral. JOHN E. MURRAY, JR., MURRAY ON

CONTRACTS § 92 [E] (5th ed. LEXISNEXUS 2011). As a general rule, a unilateral mistake does

not render a contract voidable. Rusiski v. Pribonic, 515 A.2d 507, 511 (Pa.1986). Section 153 of

the Restatement (Second) of Contracts sets forth the requirements for when a unilateral mistake

makes a contract voidable. Section 153 states:

Where a mistake of one party at the time a contract was made as to a basic

assumption on which he made the contract has a material effect on the agreed

exchange of performances that is adverse to him, the contract is voidable by him

if he does not bear the risk of the mistake under the rule stated in § 154, and

(a) the effect of the mistake is such that the enforcement of the contract

would be unconscionable, or

(b) the other party had reason to know of the mistake or his fault caused

the mistake.

RESTATEMENT (SECOND) OF CONTRACTS § 153 (Am. Law Inst. 1981); see also Lapio v.

Robbins, 729 A.2d 1229 (Pa. Super. 1999).

In this case, the mistake was clearly unilateral. The facts state that Walt, acting as

Sarah’s agent, was the sole party mistaken about the price of the coin.2 The facts state that Walt

misread the handwriting in his notebook and erroneously marked and sold the coin for $10,000

instead of $100,000 as listed on social media, the flyers, and in Walt’s own notebook of prices.

Even though the mistake here is unilateral, Sarah’s suit to rescind the sale of the coin has

a strong likelihood of success. The unilateral mistake clearly relates to a basic assumption on

which the contract was made, namely, the price of the coin. Further, the mistake materially

affects the agreed exchange of performances in a way that is adverse to Sarah because she is

receiving $90,000 less than she was supposed to receive because of Walt’s mistake.

Section 154 of the Restatement (Second) of Contracts outlines when a party to a contract

bears the risk of a mistake. Section 154 states:

2 Even if the agreement between Jed and Susan is within the ambit of the UCC, the principles of law applicable to

mistake would still apply. See 13 Pa.C.S.A. § 1103(b).

40

Page 45: OCTOBER 2020 PENNSYLVANIA BAR EXAMINATIONQuestion No. 1 Susan and Doug are a married couple who reside in Steel Town, Pennsylvania. They have been married for 25 years and have one

A party bears the risk of a mistake when

(a) the risk is allocated to him by agreement of the parties, or

(b) he is aware, at the time the contract is made, that he has limited

knowledge with respect to the facts to which the mistake relates but treats

his limited knowledge as sufficient, or

(c) the risk is allocated to him by the court on the ground that it is

reasonable in the circumstances to do so.

Id..

In this case, the facts do not support an allocation of the risk of the mistake to Sarah

based upon the factors in section 154 of the Restatement. The facts state that the agreement

signed by the parties failed to contain any mention of who bore the risk of mistake arising from

the sale of the coin. Thus, there is no allocation of the risk of mistake to Sarah by agreement of

the parties under section 154(a) of the Restatement. Additionally, there are no stated facts that

would support an allocation of the risk based upon section 154(b) and (c) of the Restatement.

Finally, the facts expressly state that Jed knew that Walt had made a mistake about the

price of the coin. As stated in the facts, Jed saw on social media that the coin was for sale for

$100,000 and he raised the amount of the advertised purchase price to buy the coin. The facts

further state that Jed was aware of Walt’s error in marking the sale price of the coin at $10,000

instead of $100,000. Based upon these facts, Jed impermissibly attempted to take advantage of

Walt’s mistake to Sarah’s detriment by snapping up the offer to buy the coin for $90,000 less

than the price that he knew was the intended price for which the coin was to be offered for sale.

See JOHN E. MURRAY, JR., MURRAY ON CONTRACTS § 92 [E] (5th ed. LEXISNEXUS 2011).

Although a unilateral mistake generally does not render a contract voidable by the

mistaken party, Sarah’s suit to rescind the sale of the 1943 Copper Penny to Jed likely will be

successful because Jed, who knew that Walt had erroneously marked the price of the coin at

$10,000 instead of $100,000, cannot “snap up” the unilateral mistake.

4(a). Because Jed and Mary hold title to Xanadu as tenants by the entireties, Jed cannot

unilaterally transfer his interest in the property to Foster-Kane & Co. (FKC).

The facts state that Jed and Mary owned Xanadu as tenants by the entireties.

“Historically, a tenancy by the entireties ‘is a form of co-ownership in real and personal property

held by a husband and wife with right of survivorship.’” In Re Estate of Navarra, 113 A.3d 829,

832 (Pa. Super. 2015) (citation omitted). A tenancy by the entireties is predicated upon the legal

unity of husband and wife as one person or unit. Beihl v. Martin, 84 A. 953, 954 (Pa. 1912). “Its

essential characteristic is that each spouse is seized per tout et non per my, i.e., of the whole or

the entirety and not of a share, moiety, or divisible part.” In Re Gallagher’s Estate, 43 A.2d 132,

133 (Pa. 1945) (citations omitted).

41

Page 46: OCTOBER 2020 PENNSYLVANIA BAR EXAMINATIONQuestion No. 1 Susan and Doug are a married couple who reside in Steel Town, Pennsylvania. They have been married for 25 years and have one

“It is well settled in this Commonwealth that where there exists an estate by the entireties

in real property, neither spouse, acting independently, may dispose of any portion so as to work a

severance of the estate, nor encumber the property in any way.” Borrello v. Lauletta, 317 A.2d

254, 255 (Pa. 1974) (citations omitted). Further, there is “no general agency arising from the

marital relationship nor any presumption flowing therefrom that either spouse has authority to

convey real estate held by the entireties without the other’s joinder therein.” Id.

Because Jed and Mary owned Xanadu as tenants by the entireties, Jed cannot unilaterally

transfer his interest in the property to FKC.

4(b). Upon their divorce, the tenancy by the entireties in Xanadu between Jed and Mary

would be severed resulting in each owning an undivided one-half interest in Xanadu

as tenants in common. As a tenant in common, Jed has the power to transfer his

interest in Xanadu to FKC without Mary’s consent. Therefore, following the

divorce and Jed’s unilateral conveyance of his interest, Mary and FKC are the

owners of undivided one-half interests in Xanadu as tenants in common.

“Because a tenancy by the entireties is grounded in the unity of the marital relationship, it

can be severed only in certain limited circumstances.” Clingerman v. Sadowski, 519 A.2d 378,

381 (Pa. 1986). One of those circumstances causing the legal unity of person existing in a

tenancy by the entireties to be destroyed is the divorce of the parties. Id. The general rule set

forth in section 3507 of the Pennsylvania Divorce Code is that a divorce severs the tenancy by

the entireties and the resulting joint interest of the parties in the property becomes a tenancy in

common. 23 Pa.C.S.A. § 3507(a).

“[A] tenancy in common is an estate in which there is unity of possession but separate

and distinct titles. In Re Estate of Quick, 905 A.2d 471, 474 (Pa. 2006). In a tenancy in

common, each tenant owns the whole of the undivided interest. 1 RONALD M. FRIEDMAN,

LADNER PENNSYLVANIA REAL ESTATE LAW, § 8.03 (6th ed. George T. Bisel Co., Inc. 2013).

“Thus, a tenant in common may, without the consent of his cotenant, sell, convey or dispose of

his undivided interest in the property.” Werner v. Quality Serv. Oil Co., Inc., 486 A.2d 1009,

1012 (Pa. Super. 1984).

Upon their divorce, the tenancy by the entireties in Xanadu between Jed and Mary was

severed and resulted in the creation of a tenancy in common. As a tenant in common, Jed would

have the right to convey his interest in Xanadu to FKC without Mary’s consent. Therefore,

following the divorce and Jed’s unilateral conveyance, Mary and FKC are the owners of

undivided one-half interests in Xanadu as tenants in common.

42

Page 47: OCTOBER 2020 PENNSYLVANIA BAR EXAMINATIONQuestion No. 1 Susan and Doug are a married couple who reside in Steel Town, Pennsylvania. They have been married for 25 years and have one

Question No. 5 - Grading Guidelines

1. Fee Simple Subject to a Condition Subsequent and Power of Termination/Right of

Re-entry

Comments: Candidates should correctly identify the applicable interests as a fee simple subject

to a condition subsequent and a power of termination or right of re-entry. Candidates should

discuss what language is needed to create a fee simple subject to a condition subsequent and the

rights associated with a power of termination or right of re-entry. Candidates also should

recognize that a holder of a power of termination/right of re-entry does not automatically obtain

title upon the happening of the event stated in the condition. Rather, the holder of the power of

termination must take some action to retake or perfect title.

6 Points

2. Third Party Beneficiary of a Contract

Comments: The candidates should recognize that under certain circumstances a contract can be

enforced by a person who is not a party to the agreement. The candidates should discuss the

elements necessary for third party beneficiary status and apply these elements to the stated facts.

4 Points

3. Unilateral Mistake in a Contract

Comments: Candidates should discuss the general principles of law governing contractual

mistake and the limited circumstances under which relief will be granted for a unilateral mistake.

Based upon the applicable legal criteria, candidates should develop well-reasoned arguments to

support a rescission of a contract based upon the unilateral mistake stated in the facts.

5 Points

4. Severance of a Tenancy by the Entireties

Comments: Candidates should recognize the well-settled principle in Pennsylvania law that

neither spouse acting unilaterally may dispose of any interest in property held as tenants by the

entireties. Additionally, candidates should recognize that a divorce severs the tenancy by the

entireties and results in a tenancy in common with each co-tenant owning an undivided one-half

interest in the property and the right to convey the tenant’s interest without the consent of the

other tenant. Candidates should apply these principles in determining the ownership of the

property in question.

5 Points

43

Page 48: OCTOBER 2020 PENNSYLVANIA BAR EXAMINATIONQuestion No. 1 Susan and Doug are a married couple who reside in Steel Town, Pennsylvania. They have been married for 25 years and have one

Question No. 6

Dave’s Market (Dave’s) is a small convenience store located in D County, Pennsylvania,

near the border of P County, Pennsylvania. Dave’s owner is Dave, a sole proprietor, who resides

in an apartment above the store. Dave never travels outside D County. Dave’s is located on a

well-traveled road and is open from 6:00 a.m. to midnight, seven days a week.

Dave also owns the real property on which his store and its parking lot are situated. The

only public entrance to Dave’s store is at the front of the store. There is a sidewalk directly in

front of the store that borders Dave’s blacktop-paved parking lot. Customers enter the parking

lot via the designated public entrance. Dave’s parking lot in front of his store has parking spaces

located along the front of the sidewalk in front of the building. Customers of Dave’s park in the

parking spots with their vehicles facing the sidewalk that runs along the storefront. There is a

no-parking area directly in front of the public entrance to the store to allow for entry and exit by

Dave’s customers. As Dave is aware, however, customers typically exit their vehicles and step

right onto the sidewalk that runs along the storefront rather than walking across the parking lot

behind other parked vehicles to reach the designated public entrance area.

Dave has had a problem with customers pulling their cars too far forward when they park

in the provided parking spaces, thereby blocking the sidewalk that runs along the storefront. To

prevent the sidewalk from being blocked, Dave recently had his paving contractor place a 6-inch

blacktop curb along the sidewalk in front of the parking spaces. The 6-inch blacktop curbing is

not marked. Several customers have commented to Dave that the blacktop curbing is virtually

invisible at night given the lighting that exists outside of Dave’s store.

Pete, a resident of P County, Pennsylvania, stopped at Dave’s Market at 11 p.m. while

traveling through D County. Pete parked in Dave’s parking lot in a space along the front of

Dave’s, exited his vehicle, began walking to the sidewalk in front of his car, and tripped on the

44

Page 49: OCTOBER 2020 PENNSYLVANIA BAR EXAMINATIONQuestion No. 1 Susan and Doug are a married couple who reside in Steel Town, Pennsylvania. They have been married for 25 years and have one

blacktop curbing, which he failed to see. Pete sustained a broken ankle and broken wrist that

required hospital treatment and an overnight stay in the hospital.

Pete consulted with his attorney regarding the incident at Dave’s. Pete’s attorney has

timely filed a civil complaint against Dave in the P County Court of Common Pleas. Pete’s

attorney has retained an expert witness, Whitman, who will testify at trial concerning proper

safety measures for public parking lots and safety measures or the lack thereof at Dave’s

parking lot when Pete fell and was injured. A separate witness, Lumen, a technician who

measured the nighttime illumination of Dave’s parking lot, will testify concerning his technical

measurements. Because Lumen will be unavailable until the last day of trial, Whitman will

testify first. Pete’s attorney plans to base his questions to Whitman on assumptions that the

nighttime illumination of Dave’s parking lot was as measured by Lumen.

1. For this question only, assume the facts above are admissible. What tort cause of

action should Pete assert against Dave, and what is Pete’s likelihood of success?

2. When Dave first met with his attorney following service of Pete’s complaint, his

attorney formed an opinion that Pete should have filed the complaint in D County

instead of P County. Under the Pennsylvania Rules of Civil Procedure, how and

when must Dave’s counsel raise this issue; assuming the issue is properly raised,

will Dave’s counsel be successful; and what remedy would apply if Dave were to

prevail on this issue?

3. Assume Whitman is qualified as an expert and that his testimony is relevant.

Under the Pennsylvania Rules of Evidence, what must Pete’s attorney establish so

that his hypothetical questions to Whitman based on assumptions about the

nighttime outdoor lighting at Dave’s will be allowed by the court if Dave’s counsel

objects?

45

Page 50: OCTOBER 2020 PENNSYLVANIA BAR EXAMINATIONQuestion No. 1 Susan and Doug are a married couple who reside in Steel Town, Pennsylvania. They have been married for 25 years and have one

Question No. 6: Examiner’s Analysis

1. Pete should assert a negligence claim against Dave and will likely succeed.

Pete should assert a negligence case against Dave for injuries sustained on Dave’s

premises. In a negligence case the plaintiff must prove the following four elements to state a

colorable cause of action: (1) the defendant owed the plaintiff a duty of care recognized by the

law; (2) the defendant breached his duty of care to the plaintiff; (3) the defendant’s breach

caused the resulting injury; and (4) actual harm occurred to the plaintiff and damages were

sustained. Grove v. Port Auth., 218 A.3d 877, 889 (Pa. 2019). That duty of care is dependent on

Pete’s status in relation to Dave.

Pete’s status while on Dave’s property was that of an invitee (he was not trespassing on

Dave’s property) as Dave implicitly invited Pete and other customers onto the property by being

open for business. The Restatement (Second) of Torts defines an “invitee” as follows:

§ 332 Invitee Defined

(1) An invitee is either a public invitee or a business visitor.

(2) A public invitee is a person who is invited to enter or remain on land as a

member of the public for a purpose for which the land is held open to the public.

(3) A business visitor is a person who is invited to enter or remain on land for a

purpose directly or indirectly connected with business dealings with the possessor

of the land.

RESTATEMENT (SECOND) OF TORTS, § 332 (Am. Law. Inst. 1965). See Cagey v. Commonwealth,

179 A.3d 458, 465 n.5 (Pa. 2018) (citing § 332 as Pennsylvania law). Pete was on Dave’s

property with the intent of entering Dave’s as a customer. Dave owed Pete a duty of care as an

invitee.

The Restatement (Second) of Torts explains the duty owed to an invitee:

§ 343 Dangerous Conditions Known to or Discoverable by Possessor

A possessor of land is subject to liability for physical harm caused to his invitees

by a condition of the land if, but only if, he

(a) knows or by the exercise of reasonable care would discover the condition, and

should realize that it involves an unreasonable risk of harm to such invitees, and

(b) should expect that they will not discover or realize the danger, or will fail to

protect themselves against it, and

(c) fails to exercise reasonable care to protect them against the danger.

RESTATEMENT (SECOND) OF TORTS, § 343 (Am. Law Inst. 1965). See Funari v. Valentino, 257

A.2d 259, 260 (Pa. 1969) (applying § 343). Similarly, Pennsylvania law has long provided that

an owner owes a duty to invitees to keep the premises in reasonably safe condition or warn of

46

Page 51: OCTOBER 2020 PENNSYLVANIA BAR EXAMINATIONQuestion No. 1 Susan and Doug are a married couple who reside in Steel Town, Pennsylvania. They have been married for 25 years and have one

any dangers of which the owner is or should be aware. See, e.g., Polinelli v. Union Supply Co.,

170 A.2d 351, 354 (Pa. 1961); see also Kerwood v. Rolling Hill Corp., 225 A.2d 918, 920 (Pa.

1967) (invitee is owed a duty of reasonable care to provide a reasonably safe entry).

Here, Pete was a customer of Dave’s business and consequently an invitee. Dave,

therefore, owed Pete a duty of reasonable care to make the premises safe, provide a safe entry to

the premises, and warn of any dangers of which Dave was or should have been aware. The facts

indicate the blacktop curbing was not visible to patrons at night and that several patrons had

informed Dave of that fact. Dave therefore had a duty to Pete either to make the curbing

reasonably safe (for example by marking it with bright or reflective paint) or to warn patrons

such as Pete of the presence of the black curbing. The facts suggest Dave breached his duty,

inasmuch as there are no facts indicating that Dave took any corrective action after being

informed of the hazard by several customers. A jury could reasonably find that Pete’s trip and

fall was caused by Dave’s breach of care, resulting in Pete’s failure to see the curbing. The facts

indicate Pete sustained harm and damages. The facts support all four elements of a negligence

action (duty, breach, causation, and damages). Therefore, Pete is likely to succeed in a

negligence action against Dave.

2. Dave’s counsel should challenge venue by preliminary objection and the action should

be transferred to D County.

In general, Pennsylvania’s venue rule provides that “an action against an individual may

be brought in and only in a county in which (1) the individual may be served or in which the

cause of action arose or where a transaction or occurrence took place out of which the cause of

action arose or in any other county authorized by law . . . .” Pa.R.C.P. No. 1006(a)(1). Dave

lives and works in D County and never travels outside D County. There is nothing in the facts

suggesting he may be served in P County. Pete fell in Dave’s parking lot in D County. Thus, the

cause of action arose in D County and did not arise in P County. The facts do not suggest that

there is any law otherwise authorizing venue in P County. Therefore, venue is not proper in P

County, and Dave’s venue challenge should be successful.

Pennsylvania Rule of Civil Procedure 1006(e) provides, “[i]improper venue shall be

raised by preliminary objection and if not so raised shall be waived.” Pa.R.C.P. No. 1006(e). If

Dave believes venue in P County is improper, he must raise that issue in a preliminary objection

to Pete’s complaint. Because improper venue is a waivable objection, if Dave does not properly

challenge it in a preliminary objection, his challenge to improper venue will be waived and the

action will proceed in P County. See Zappala v. Brandolini Prop. Mgmt., 909 A.2d 1272, 1286

(Pa. 2006) (preliminary objection is the exclusive method for challenging improper venue; if not

raised by preliminary objection, the challenge is waived).

“If a preliminary objection to venue is sustained and there is a county of proper venue

within the State the action shall not be dismissed but shall be transferred to the appropriate court

of that county . . . .” Pa.R.C.P. No. 1006(e). Because venue does not properly lie in P County,

Dave’s preliminary objection to venue should be sustained.

47

Page 52: OCTOBER 2020 PENNSYLVANIA BAR EXAMINATIONQuestion No. 1 Susan and Doug are a married couple who reside in Steel Town, Pennsylvania. They have been married for 25 years and have one

Dave both resides and does business in D County and thus may be served there. See

Pa.R.C.P. No. 402(a)(2)(i) (iii) (a party may be served at his residence or place of business).

Because Pete fell in Dave’s parking lot, the incident arose at Dave’s, which is located in D

County. Therefore, under Pa.R.C.P. No. 1006(a)(1), D County is a proper venue for Pete’s

action. Accordingly, the trial court should not dismiss Pete’s complaint, but rather, should

transfer the action from P County to D County.

3. A qualified expert witness may give a relevant opinion in response to hypothetical

questions based on assumed facts, provided that before the conclusion of the party’s

case, the assumed facts are established by competent record evidence.

An expert witness is permitted to respond to a hypothetical question with an

opinion provided the facts are supported by competent evidence at some point in

the trial. However, the court may properly disallow a hypothetical question if the

hypothetical is not supported by sufficient evidence. The expert may not base an

opinion merely on conjecture.

LEONARD PACKEL AND ANNE BOWEN POULIN, EVIDENCE, 1 West’s Pa. Prac., Evidence §703-1

(4th ed.) (2019). Pennsylvania Rule of Evidence 703 states:

An expert may base an opinion on facts or data in the case that the expert has

been made aware of or personally observed. If experts in the particular field

would reasonably rely on those kinds of facts or data in forming an opinion on

the subject, they need not be admissible for the opinion to be admitted.

Pa.R.E. 703.

Expert witnesses are frequently made aware of facts or data by being asked to assume

those facts or data through the use of hypothetical questions. “The use of hypothetical questions

is proper when there is evidence of record supporting the hypothetical.” Commonwealth v.

Galvin, 985 A.2d 783, 801 (Pa. 2009) (citing Commonwealth v. Petrovich, 648 A.2d 771, 772

(Pa. 1994) (expert may give an opinion based on a hypothetical question if the set of facts

assumed in the hypothetical question is supported by competent evidence).

Here, Pete’s expert witness, Whitman, will be asked to opine about what safety measures

Dave should have taken regarding his parking lot. Whitman will be asked to assume a

hypothetical set of facts concerning the lighting conditions of the parking lot. The assumed facts

will be supported by testimony to follow Whitman’s testimony to be provided by the technician,

Lumen. Pete’s attorney should disclose the proposed testimony of Lumen to the court and ask

that he be permitted to ask the hypothetical questions based upon that expected testimony.

Provided Pete’s attorney offers competent evidence during the trial that supports the facts

Whitman is asked to assume, Whitman’s opinion will be proper and should be admitted in

evidence.

48

Page 53: OCTOBER 2020 PENNSYLVANIA BAR EXAMINATIONQuestion No. 1 Susan and Doug are a married couple who reside in Steel Town, Pennsylvania. They have been married for 25 years and have one

Question No. 6: Grading Guidelines

1. Torts – Premises Liability

Comments: The applicant should recognize that the proper cause of action sounds in negligence.

The applicant should discuss and apply each element – duty, breach, causation, and damages – to

the facts as given, including a discussion of the standard of care that is due a business invitee.

The applicant should conclude that the cause of action is viable.

9 points

2. Civil Procedure – Venue

Comments: The applicant should recognize that the issue is one of proper venue. The applicant

should recognize that improper venue must be challenged by preliminary objection. The

applicant should apply the venue rule to the facts as given and should conclude that the venue

challenge will be successful if properly raised, and that the appropriate action by a court is not to

dismiss the action, but to transfer it to the correct venue.

6 points

3. Evidence – Hypothetical Questions to Expert Witness

Comments: The applicant should discuss the permissibility of hypothetical questions to experts

and the requirements for establishing by proper evidence the facts that an expert is asked to

assume.

5 points

49

Page 54: OCTOBER 2020 PENNSYLVANIA BAR EXAMINATIONQuestion No. 1 Susan and Doug are a married couple who reside in Steel Town, Pennsylvania. They have been married for 25 years and have one

PT

Supreme Court of Pennsylvania

Pennsylvania Board of Law Examiners

Pennsylvania Bar Examination

February 24 and 25, 2015

PERFORMANCE TEST

February 24, 2015

Use GRAY covered book for your answer to the Performance Test.

© 2015 Pennsylvania Board of Law Examiners

PLACE BAR CODED APPLICANT LABEL HERE

Question Number 3

on Examplify

50

Page 55: OCTOBER 2020 PENNSYLVANIA BAR EXAMINATIONQuestion No. 1 Susan and Doug are a married couple who reside in Steel Town, Pennsylvania. They have been married for 25 years and have one

PERFORMANCE TEST

Table of Contents

FILE

1. Assignment Memorandum ..................................................................................................................53

2. Intake Memorandum ...........................................................................................................................54

3. Guidelines for Preparing Internal Legal Memorandum ......................................................................57

LIBRARY

1. 42 Pa. C.S. § 8501 ...............................................................................................................................59

2. 42 Pa. C.S. § 8541 ...............................................................................................................................59

3. 42 Pa. C.S. § 8542 ...............................................................................................................................59

4. 42 Pa. C.S. § 8545 ...............................................................................................................................60

5. 42 Pa. C.S. § 8546 ...............................................................................................................................60

6. 42 Pa. C.S. § 8549 …………………………………………………………………………………..60

7. 42 Pa. C.S. § 8550 ...............................................................................................................................60

8. 42 Pa. C.S. § 8553 ...............................................................................................................................61

9. Stevenson v. Economy Bank of Ambridge, 197 A.2d 721 (Pa. 1964) .................................................62

10. Fernandez v City of Pittsburgh, Adkins v. City of Pittsburgh, 643 A.2d 1176 (Pa. Commw. 1994) .67

51

Page 56: OCTOBER 2020 PENNSYLVANIA BAR EXAMINATIONQuestion No. 1 Susan and Doug are a married couple who reside in Steel Town, Pennsylvania. They have been married for 25 years and have one

FILE

52

Page 57: OCTOBER 2020 PENNSYLVANIA BAR EXAMINATIONQuestion No. 1 Susan and Doug are a married couple who reside in Steel Town, Pennsylvania. They have been married for 25 years and have one

ASSIGNMENT MEMORANDUM

TO: Applicant

FROM: Lilly Benjamin, Managing Partner

RE: Luke Anderson’s Watch, File #: 06031995

DATE: October 5, 2020

Our client, Luke Anderson, won a rare and valuable watch in a nationwide drawing. The

superintendent of the school district Mr. Anderson’s son attends is presently holding the watch.

Mr. Anderson is seeking our help to force the school district to return the watch. I think we

should bring an action against the school district and the superintendent, who was involved with

taking possession of the watch, for the intentional tort of conversion and should seek payment of

the full value of the watch ($750,000). As you know, the intentional torts do not include

negligence; as you will see, the relevant actions by the district superintendent were intentional

acts, not negligent acts. I am also looking into other potential claims we may be able to include.

With any luck, once we file suit they will simply return the watch to Mr. Anderson. If not, our

client wants us to pursue a judgment for the value of the watch.

Your assignment is to review the documents in the accompanying File and Library and to draft

an Internal Memorandum to me setting forth whether our client has a valid claim for conversion

and whether he can successfully bring an action in conversion against the school district and the

superintendent, only in his official capacity, to recover the full value of the watch. You should

include an analysis of any governmental immunity issues in your Internal Memorandum. Please

provide a reasoned analysis of each issue, along with the rationale used in reaching your

conclusions. Regardless of whether you believe Mr. Anderson will prevail on a claim for the

intentional tort of conversion, please include an analysis of the amount Mr. Anderson could

recover if he were to prevail, in case I include other claims.

Included in the File is my intake memorandum from my meeting with Mr. Anderson and a memo

concerning our firm’s requirements for drafting an Internal Memorandum. Included in the

Library are two cases and several statutes that will help with preparing the Internal

Memorandum. You should assume that the cases and statutes correctly set forth current

Pennsylvania law. You should also only use the facts contained in the File and the cases and

statutes contained in the Library for your legal analysis and conclusions. Do not rely upon your

personal knowledge of these issues or on rules, cases, or statutes that are not included in the

Library. Instead, you should base your analysis and conclusions solely upon the documents

provided in the File and the Library.

53

Page 58: OCTOBER 2020 PENNSYLVANIA BAR EXAMINATIONQuestion No. 1 Susan and Doug are a married couple who reside in Steel Town, Pennsylvania. They have been married for 25 years and have one

INTAKE MEMORANDUM

TO: Applicant

FROM: Lilly Benjamin, Managing Partner

RE: Luke Anderson’s Watch, File #: 06031995

DATE: October 4, 2020

Our client, Luke Anderson, a resident of Midtown, Pennsylvania, was the lucky winner of a rare,

diamond-encrusted watch given away as part of a nationwide drawing held by a watch

manufacturer’s trade association. Mr. Anderson received the watch, valued at $750,000, at the

live drawing on August 10, 2020.

Mr. Anderson is not a wealthy man and winning such a valuable prize is a life-changing event.

He and his twelve-year-old son, Joseph, took turns wearing the watch at home the first day they

had it. Mr. Anderson told his son he could wear the watch occasionally before they sell it, but

that they had to use discretion about when and where to wear it. Mr. Anderson was concerned

about keeping such a valuable object at his home, particularly because a local news station ran a

story about his good fortune. He could not afford to take out an insurance policy on the full value

of the watch, but was able to secure a policy of $100,000 in the event the watch were damaged or

stolen or if Mr. Anderson were otherwise deprived of its ownership. In an attempt to safeguard

his new watch, Mr. Anderson purchased a small safe that he could hide in his closet. Mr.

Anderson enlisted Joseph’s assistance to bolt the safe to the floor so that no one could easily

remove it. After locking the watch in the safe, Mr. Anderson hid the safe’s combination number

in his dresser drawer under some clothes.

On August 18, 2020, Joseph began his school year. That day, Joseph went to his school, located

in the Midtown School District in Midtown, Pennsylvania. Joseph encountered many students

and teachers asking about the watch. Joseph was happy to describe the watch, and he told

students his father said he could wear it whenever he wanted to. One student said if that were

true, Joseph should wear the watch to school. Joseph said he did not want to wear it to school.

Another student ridiculed Joseph for refusing to bring the watch to school.

On August 19, Joseph was so upset by his classmate’s ridicule that he snuck into his father’s

dresser, found the combination to the safe, removed the watch, and wore it to school. Joseph was

very cautious with the watch and did not tell anyone he was wearing it until his second class of

the day when he saw the classmate who ridiculed him the day before. Upon seeing that

classmate, Joseph removed his jacket, exposing the watch on his wrist, and said, “See, I can wear

it whenever I want to.” This confrontation upset Joseph’s classmate, who then tried to grab the

watch from Joseph’s wrist. The Midtown School District Superintendent, Mark Tierney, whose

office is located within Joseph’s school, happened to be walking by at the time of the

confrontation. He separated the two students and said to Joseph, “You’d better give me that

watch to put in my office for safekeeping until it is time to go home.” Joseph allowed

Superintendent Tierney to remove the watch from his arm. Superintendent Tierney told Joseph

he would put the watch in his desk drawer until the end of the day so that nothing would happen

54

Page 59: OCTOBER 2020 PENNSYLVANIA BAR EXAMINATIONQuestion No. 1 Susan and Doug are a married couple who reside in Steel Town, Pennsylvania. They have been married for 25 years and have one

to it, and he told Joseph to come get the watch at the end of the day to bring it home. As he was

walking away from Joseph, Superintendent Tierney said, “This is a gorgeous watch. I wish I had

won it.”

In early July 2020, Midtown School District performed environmental tests in Joseph’s school.

The results verified the existence of asbestos containing materials (ACM), which can be harmful

to humans who breathe in particles from the ACM. Superintendent Tierney is the chief officer

and employee in the school district and is responsible for the operation of all the schools in the

district. The Midtown School Board gave Superintendent Tierney total authority to take

whatever action he felt was necessary regarding the ACM in the school and any related or

consequential matter. Superintendent Tierney compared the test results to Environmental

Protection Agency (EPA) guidelines, and he decided the test results did not warrant immediate

remediation, which would have delayed the school year; instead, he decided the school district

would wait until the students were out of the building during the winter break to remove the

ACM. Superintendent Tierney made a public announcement that the building was safe for

occupancy and that he had scheduled remediation for the winter break, which coincides with the

last two weeks of December 2020. Despite complaints from teachers, staff, and parents about

the delay in performing remediation of the ACM, Superintendent Tierney did not change the

dates for the remediation.

Unexpectedly, however, and without any change in the building’s condition or the EPA

guidelines since the testing in early July, at lunchtime on August 19, Superintendent Tierney

instructed the students and faculty to immediately leave the building and assemble outside in

the athletic field next to the school. Superintendent Tierney informed everyone he was closing

the school building due to asbestos contamination (the ACM) and that the staff was contacting

all of the student parents to come pick up their children from the athletic field. He announced

that no one could enter the building until it was cleared by environmental consultants. To the

best of Joseph’s knowledge, the watch remained in Superintendent Tierney’s office.

Joseph’s father picked him up from school that day and Joseph confessed to wearing the watch to

school and told his father what transpired that day. Mr. Anderson immediately telephoned

Superintendent Tierney, leaving a voicemail requesting the return of his watch. Superintendent

Tierney did not return his call. The next day, Mr. Anderson again requested the return of his

watch via an e-mail message to Superintendent Tierney, who responded via e-mail message

stating, “Your watch is safe and is in my desk at my office. However, I believe the building is

unfit for human entry based on the advice of environmental consultants, so no one can enter the

building to retrieve personal belongings; not you, not the teachers, and not any of the 900

students.” Mr. Anderson has telephoned Superintendent Tierney on a daily basis to demand the

return of his watch, but Superintendent Tierney has not returned any of Mr. Anderson’s

telephone calls. Mr. Anderson has also left voicemail messages for, and sent e-mail messages to,

Superintendent Tierney offering to hire a certified environmental technician (Technician) of the

school district’s choosing, explaining that the Technician would wear the appropriate gear to

safely enter the building and retrieve the watch. Superintendent Tierney finally responded to

55

Page 60: OCTOBER 2020 PENNSYLVANIA BAR EXAMINATIONQuestion No. 1 Susan and Doug are a married couple who reside in Steel Town, Pennsylvania. They have been married for 25 years and have one

one of the e-mail messages stating, “Until the district’s environmental consultant clears the

building for human occupancy, no one will be allowed inside the school building. Further

allowing your environmental technician to retrieve your watch could expose the district to

additional liability from a student or teacher who later claims something is missing after the

technician entered the building, or to claims of unfair treatment by allowing one person to

retrieve his belongings, but not all others.” All other voicemail messages and e-mail messages

have been unanswered by Superintendent Tierney.

No one has been allowed inside the school since August 19. Five days after they evacuated the

school, students began attending classes in a nearby building that the superintendent said was

necessary to get the children back to school, as the original school building was unsafe. During a

handful of television interviews, Superintendent Tierney said he did not know if or when the

district would have the funds available to perform the necessary asbestos remediation (ACM

removal) and explained that the remediation had to be done before people could safely be allowed

to return to the school building. Superintendent Tierney also said he decided to close the school

because the environmental consultant who wrote the report on the ACM in the school building

contacted him after the school opened and encouraged him to close the building. Superintendent

Tierney stated that in light of the telephone call from the environmental consultant, he again

reviewed the environmental test results and believes he initially misunderstood the significant

levels of asbestos in the building and determined that no one would be safe in the building until

the ACM was removed. He stated he arrived at this conclusion after he reviewed federal

environmental statutes and looked again at the EPA guidelines. He said that he concluded that

a statute authorized him to close the school because closing the school was necessary to protect

the health and safety of the students and faculty (he has failed to identify the specific statute).

He additionally stated, the remediation would cost so much that the school would likely be

locked down through the end of the school year and the school may never be reopened.

Mr. Anderson came to us for help in getting his watch back. Although he has no direct evidence

that Superintendent Tierney closed the school in order to keep the watch, one of Mr. Anderson’s

friends said he saw Superintendent Tierney out at a restaurant one night in September 2020

with something very shiny on his wrist. He believes Superintendent Tierney has the watch and

has no plans to return it and is using the presence of a so-called environmental issue to cover up

the fact that he does not plan to return the watch. He also believes that, if the school ever

reopens, Superintendent Tierney will claim the watch is no longer in his desk and that someone

must have stolen it while the school was closed for environmental reasons.

Mr. Anderson asked the firm for assistance with getting his watch back, or, in the alternative,

obtaining the full value of the watch.

56

Page 61: OCTOBER 2020 PENNSYLVANIA BAR EXAMINATIONQuestion No. 1 Susan and Doug are a married couple who reside in Steel Town, Pennsylvania. They have been married for 25 years and have one

FORMATTING MEMORANDUM FOR DRAFTING INTERNAL MEMORANDA

TO: All Staff

FROM: Lilly Benjamin, Managing Partner

RE: Guidelines and Format for Preparation of Internal Memoranda

DATE: October 1, 2020

Use the following guidelines and format for preparing all internal legal memoranda:

1. The document should be entitled “Memorandum of Law.”

2. At the top of the memorandum, include a heading similar to the heading above (e.g. – To,

From, Re, and Date). In the “From” section, state only the word “Applicant;” do not include

your name.

3. Include a brief introductory paragraph laying out the purpose of the memorandum.

4. The memorandum should be divided into sections, one for each issue discussed, where

appropriate. Each section should begin with a short heading that reflects the issue being

addressed.

5. Each section should also include a reasoned analysis supporting your conclusion. Identify the

relevant and controlling legal principles and apply these legal principles to the facts to

demonstrate the reasoning that supports your conclusion on the issue presented. If there are

facts and legal principles relevant to any point or element in your analysis that could be

argued to support a different conclusion, identify and discuss those principles or facts.

6. Include all relevant facts needed to resolve the issue(s) presented as well as any background

facts helpful to understanding the issues.

7. State your conclusion(s) as a positive statement that responds to the question(s) raised by the

issue presented.

8. Bluebook citations are not necessary; however, you must include sufficient informal citations

to the appropriate authority (legal or otherwise), such that the reader will know to which

document you are referring.

57

Page 62: OCTOBER 2020 PENNSYLVANIA BAR EXAMINATIONQuestion No. 1 Susan and Doug are a married couple who reside in Steel Town, Pennsylvania. They have been married for 25 years and have one

LIBRARY

58

Page 63: OCTOBER 2020 PENNSYLVANIA BAR EXAMINATIONQuestion No. 1 Susan and Doug are a married couple who reside in Steel Town, Pennsylvania. They have been married for 25 years and have one

42 Pa. C.S. § 8501. Definitions.

***

“Local Agency.” A government unit other than the Commonwealth government. The

term includes . . . [a school district]. . . .

42 Pa. C.S. § 8541. Governmental immunity generally.

Except as otherwise provided in this subchapter, no local agency shall be liable for any

damages on account of any injury to a person or property caused by any act of the local

agency or an employee thereof or any other person. . . .

42 Pa. C.S. § 8542. Exceptions to governmental immunity.

(a) Liability imposed. A local agency shall be liable for damages on account of an injury

to a person or property within the limits set forth in this subchapter if both of the

following conditions are satisfied and the injury occurs as a result of one of the acts set

forth in subsection (b):

(1) The damages would be recoverable under common law or a statute creating a

cause of action if the injury were caused by a person not having available a defense

under section 8541 (relating to governmental immunity generally) or section 8546

(relating to defense of official immunity); and

(2) The injury was caused by the negligent acts of the local agency or an employee

thereof acting within the scope of his office or duties with respect to one of the

categories listed in subsection (b). As used in this paragraph, "negligent acts" shall

not include acts or conduct which constitutes a crime, actual fraud, [or] actual

malice. . . .

(b) Acts which may impose liability. The following acts by a local agency or any of its

employees may result in the imposition of liability on a local agency:

* * *

(2) Care, custody or control of personal property. The care, custody or control of

personal property of others in the possession or control of the local agency. The only

losses for which damages shall be recoverable under this paragraph are those

59

Page 64: OCTOBER 2020 PENNSYLVANIA BAR EXAMINATIONQuestion No. 1 Susan and Doug are a married couple who reside in Steel Town, Pennsylvania. They have been married for 25 years and have one

property losses suffered with respect to the personal property in the possession or

control of the local agency. . . .

42 Pa. C.S. § 8545. Official liability generally.

An employee of a local agency is liable for civil damages on account of any injury to a

person or property caused by acts of the employee which are within the scope of his office

or duties only to the same extent as his employing local agency and subject to the

limitations imposed by this subchapter. . . .

42 Pa. C.S. § 8546. Defense of official immunity.

In any action brought against an employee of a local agency for damages on account of an

injury to a person or property based upon claims arising from, or reasonably related to, the

office or the performance of the duties of the employee, the employee may assert on his

own behalf, or the local agency may assert on his behalf:

(1) Defenses which are available at common law to the employee.

(2) The defense that the conduct of the employee which gave rise to the claim was

authorized or required by law, or that he in good faith reasonably believed the

conduct was authorized or required by law. . . .

42 Pa. C.S. § 8549. Limitation on Damages.

In any action brought against an employee of a local agency for damages on account of an

injury to a person or property in which it is judicially determined that the act of the

employee caused the injury and that such act was, or that the employee in good faith

reasonably believed that such act was, within the scope of his office or duties, damages

shall be recoverable only within the limits set forth in this subchapter.

42 Pa. C.S. § 8550. [Crime, Actual Fraud, or Actual Malice].

In any action against a local agency or employee thereof for damages on account of an

injury caused by the act of the employee in which it is judicially determined that the act of

the employee caused the injury and that such act constituted a crime, actual fraud, [or]

actual malice . . . , the provisions of sections 8545 (relating to official liability generally),

60

Page 65: OCTOBER 2020 PENNSYLVANIA BAR EXAMINATIONQuestion No. 1 Susan and Doug are a married couple who reside in Steel Town, Pennsylvania. They have been married for 25 years and have one

8546 (relating to defense of official immunity) . . . and 8549 (relating to limitation on

damages) shall not apply.

42 Pa. C.S. § 8553. Limitations on damages.

(a) General rule. Actions for which damages are limited by reference to this subchapter

shall be limited as set forth in this section.

(b) Amounts recoverable. Damages arising from the same cause of action or

transaction or occurrence or series of causes of action or transactions or occurrences shall

not exceed $500,000 in the aggregate.

(c) Types of losses recognized. Damages shall be recoverable only for:

***

(6) Property losses.

(d) Insurance benefits. If a claimant receives or is entitled to receive benefits under a

policy of insurance other than a life insurance policy as a result of losses for which

damages are recoverable under subsection (c), the amount of such benefits shall be

deducted from the amount of damages which would otherwise be recoverable by such

claimant. . . .

61

Page 66: OCTOBER 2020 PENNSYLVANIA BAR EXAMINATIONQuestion No. 1 Susan and Doug are a married couple who reside in Steel Town, Pennsylvania. They have been married for 25 years and have one

STEVENSON, Appellant, v. ECONOMY BANK OF AMBRIDGE, Appellee

197 A.2d 721 (Pa. 1964)

***

Appellant, Eliza W. Stevenson, instituted an action of trespass in the Court of Common

Pleas of Beaver County against appellee, Economy Bank of Ambridge, for conversion of the

contents of a safe deposit box, leased by appellee to Doctor W.B. Carson, appellant's

brother-in-law, who died testate April 17, 1961, and to appellant as co-tenants.

The complaint charged appellee-bank . . . with conversion of . . . $82,300 in cash, which

appellant claimed was her property and to which she had the right of possession under the

terms of the [safe deposit box] lease.

***

The relevant facts from which this controversy arises and on which it must be determined

are not in dispute. Decedent and his wife, for many years prior to her death (October 17,

1955), were co-lessees of the safe deposit box. On November 16, 1955, shortly after his

wife's death, Doctor Carson substituted the name of appellant, his wife's sister, for that of

his deceased wife as co-lessee. [2] The lease for the safe deposit box was signed by both

decedent and appellant and two keys were issued. Each lessee retained a key, but

appellant never exercised her right of access to the box during Doctor Carson's lifetime.

Appellant testified that at the time Doctor Carson had her sign the lease, he gave her one

of the keys. He also told her that the purpose of having her sign the lease was to make her

co-owner and that she could enter the box at any time she desired.

Decedent's will, dated September 4, 1959, was prepared by his attorney and, at decedent's

death, was in possession of the scrivener. Testator bequeathed his residence and half of

his approximately $490,000 estate (after $6000 of pecuniary legacies) to appellant and

designated his attorney as executor. On the day following decedent's death, appellant and

decedent's attorney together sought and obtained access to the safe deposit box for the sole

purpose of learning whether it contained a will of later date. No other will was found, and

the box, with its contents intact, was returned to the vault. Appellant, on this occasion,

had provided her key for entry to the box. Testator's counsel read a part of decedent's will

to appellant and informed her and the president of the bank that the safe deposit box

could not be opened until the will had been probated and that nothing could be removed

until a proper inventory of the contents had been completed. The record does not disclose

that appellant voiced any objection, nor did she express a desire to remove anything from

the box at that time.

On the next afternoon (April 19), following decedent's funeral, appellant, accompanied by

the funeral director, went to the bank and requested access to the box for the purpose of

removing the contents in order to take the cash for herself . . . . Appellant was advised by

62

Page 67: OCTOBER 2020 PENNSYLVANIA BAR EXAMINATIONQuestion No. 1 Susan and Doug are a married couple who reside in Steel Town, Pennsylvania. They have been married for 25 years and have one

the bank employee in charge of the vault that she was not permitted into the box and was

refused permission to sign an entry slip. Shortly thereafter, the named executor entered

the bank and, upon being informed of appellant's purpose, again stated that access to the

box was not to be permitted. He repeated his direction that the box should remain closed

until after probate of decedent's will and the granting of letters testamentary. It was on

the basis of these instructions that appellee-bank refused appellant entry that afternoon.

The executor advised appellant that he intended to probate the will the next day (April 20)

and requested her to meet him at the bank on the succeeding day (April 21) to open the

box and to make an inventory of its contents.

On April 20, the will was admitted to probate by the Register of Wills of Allegheny

County, and letters testamentary were duly issued to the testator's named executor.

The following day, April 21, at the appointed time, appellant and her adult son came to

the bank; present also were the president of the bank, the executor, and a representative

of the Pennsylvania Department of Revenue (whom the executor requested to be present).

[footnote omitted] Appellant furnished her key, she and the executor together signed the

entry slip, and the box was removed from the vault and opened. The contents were

inventoried and written copies of the list were signed by appellant, her son, the executor,

the bank president and the representative of the Department of Revenue.

***

[C]ash in unmarked envelopes, amounting to $82,300, was counted by all those present,

including appellant's son. To those assembled, appellant stated that decedent had told her

that she should have the money. The executor, however, denied her possession and

claimed it as an asset of the estate. Appellant could make her claim to the money before

the Orphans' Court of Allegheny County, he added. . . . Upon completion of the inventory,

the executor took possession of the entire contents of the box . . . .

***

The parties concede as, indeed, they must under prevailing statutory and decisional

authorities [footnote omitted] that the issue of title to the cash found in the safe deposit

box is exclusively within the jurisdiction of the Orphans' Court of Allegheny County. Only

that forum is legally competent to make that initial determination. Since that adjudication

is yet to be made, it is appropriate to observe that our disposition here is not intended to

influence and should not be regarded as indicating or influencing the result here to be

reached.

In this trespass action, the principal issue is not title to the cash at decedent's death, as it

would be in the orphans' court, but rather appellant's right to possession of the cash on

April 19 under the terms of the lease and whether the bank's refusal to grant her access to

63

Page 68: OCTOBER 2020 PENNSYLVANIA BAR EXAMINATIONQuestion No. 1 Susan and Doug are a married couple who reside in Steel Town, Pennsylvania. They have been married for 25 years and have one

the box two days after decedent's death constituted a conversion of the cash contents, so as

to entitle her to recover damages in that amount.

Appellant's basic contention is that the written rental lease, by its specific language, gave

her the right to enter the safe deposit box on April 19, as she requested. The lease

provided: "A rental contract, signed by two or more lessees, constitutes a separate rental

to each, and either of them shall have access, free from liability on the part of lessor for

misappropriating any of the contents thereof. . . . In all cases of joint lessees, it is hereby

declared that each of them has such interest in the entire contents of said box as to entitle

him or her to the possession thereof, without liability to lessor for misappropriating same .

. . ." It is argued that appellant, having the right of access to the box and being known to

the bank as a co-lessee, [footnote omitted] could not legally be denied access to the rented

box, and that the refusal of access constituted a conversion of the cash in the box.

Appellant further contends that the bank, in refusing her access, could not have been

motivated by a concern for possible liability arising from conflicting claims between the co-

lessees because the lease, by its express terms, provided that the lessor shall not be liable

for misappropriation of the contents by either co-tenant.

Appellee's primary contention is that the orphans' court has exclusive jurisdiction to

determine both ownership and possession of the cash and that appellant's right of access

to the box pursuant to the lease did not, in any way, fix the ownership of its contents. In

the absence of proof of appellant's ownership, the bank urges, appellant is not entitled to

judgment in the amount of the cash found in the box.

Appellee further contends that it was legally justified in deferring appellant's access to the

box for two days pending probate of the will and a proper inventory of the contents, and

that such reasonable delay did not constitute a conversion of the cash. Moreover, appellee

points out that on the second day, access to the box and to its contents were made

available to appellant and the executor and that on that occasion, appellant surrendered

the box and signed the bottom of the printed form lease. It is contended that she thereby

released the bank from any claim against it for conversion. We do not view this occurrence

or appellant's signature to the form as a waiver or release by her of any claim she may

have acquired for conversion. The transaction was merely the termination of the lease

agreement and the physical surrender of the rented box, nothing more.

***

"A [common law] conversion is the deprivation of another's right of property in, or use or

possession of, a chattel, or other interference therewith, without the owner's consent and

without lawful justification." [Citation omitted.] Prosser states that "conversion is an act

of interference with the dominion or control over a chattel. . . . Conversion may be

committed by: . . . (c) Unreasonably withholding possession from one who has the right to

it." [Citation omitted.] Since title to the cash is for determination by a tribunal other than

64

Page 69: OCTOBER 2020 PENNSYLVANIA BAR EXAMINATIONQuestion No. 1 Susan and Doug are a married couple who reside in Steel Town, Pennsylvania. They have been married for 25 years and have one

the court below, appellant's action for conversion, if it is to prevail, must be based upon

her right of access to the box and to possession of its cash contents. Was the bank, as

appellant contends, obligated under the terms of the lease to give appellant, at her

request, access to the box; or was the bank, as it urges, justified in denying her entry until

the executor qualified? This fundamental aspect of the controversy must be resolved by

resort to the provisions of the lease agreement.

The pertinent portions of the lease, already quoted, specifically and unmistakably

conferred upon appellant, as a co-lessee, the right of access to the box both before and after

her co-lessee's death. The express language "either of them shall have access" and the

further declaration that "each of them has such interest in the entire contents of said box

as to entitle him or her to the possession thereof," do not restrict or limit her right of entry

and possession to only those occasions when she is accompanied by her co-lessee or by his

personal representative in the event of his death.

Were we to accede to appellee's argument, we would, in effect, expand the language of the

lease by reading into it conditions and limitations upon the right of entry not contained in

its terms. A construction of the lease which permits lessor to deny or delay access for the

reasons asserted by it and upon grounds not recited or otherwise implied in the lease is to

add to that writing restrictions not contained in the contract itself. To do so would be an

obvious violation of the parol evidence rule. [Citation omitted.]

Careful consideration of the lease satisfies us that appellant had a contractual right of

access to and entry into the box on April 19, and that the bank was without legal authority

or justification to preclude her from exercising that right. Its refusal to admit appellant to

the box was a willful interference with her right of possession of the contents and

constituted an act of conversion. Allowing her access with the executor two days later did

not bar the action of conversion which she acquired on April 19. We conclude, therefore,

that the court below, in the light of the undisputed facts, erred in refusing appellant's

request for binding instructions.

***

NOTES

***

[2] Appellant's evidence is that she went to keep house for Doctor Carson at his residence

in Allegheny County, and at his request agreed to take care of him for the rest of his life

without wages. He, in turn, promised to leave her the house and approximately $82,000 in

cash in his safe deposit box. There is evidence also that decedent frequently told her that

the cash in the safe deposit box was hers and that she could go there and get it whenever

she desired before or after his death, and that he would not include the cash in his will

because it was hers and he had already paid taxes on it and did not want it to be subject to

65

Page 70: OCTOBER 2020 PENNSYLVANIA BAR EXAMINATIONQuestion No. 1 Susan and Doug are a married couple who reside in Steel Town, Pennsylvania. They have been married for 25 years and have one

taxes again. It was her testimony also that decedent had told her that on his death she

was to go to the bank, enter the box, remove its contents, [and] retain the money for

herself . . . .

66

Page 71: OCTOBER 2020 PENNSYLVANIA BAR EXAMINATIONQuestion No. 1 Susan and Doug are a married couple who reside in Steel Town, Pennsylvania. They have been married for 25 years and have one

FERNANDEZ v. CITY OF PITTSBURGH, Appellant.

ADKINS v. The CITY OF PITTSBURGH and Carmenza Fernandez.

643 A.2d 1176 (Pa. Commw. 1994)

***

The City of Pittsburgh appeals from the orders of the Court of Common Pleas of Allegheny

County which denied the City's post-trial motion seeking judgment notwithstanding the

verdict or a new trial, and ordered it to pay delay damages to William Adkins and

Carmenza Fernandez (collectively Plaintiffs) in their civil lawsuit against the City arising

out of an automobile accident caused by a defective condition in a road.

The issues presented on appeal are . . . whether the trial court incorrectly calculated

Plaintiffs' awards . . . .

On June 20, 1989, Fernandez was driving a car west on Second Avenue in the City of

Pittsburgh when she and Adkins, the passenger, were injured in an automobile collision.

Plaintiffs sued the City and alleged, in pertinent part, that a defective condition in the

road caused Fernandez' car to strike another vehicle. As a result of the accident, Adkins

alleged that he suffered a skull fracture, cerebral trauma with damage to his brain,

multiple trauma, ruptured spleen, assorted abrasions, lacerations and contusions, and

shock with injury and damage to his nerves and nervous system; and Fernandez alleged

she suffered injury to her pelvis, legs, arms, and head. [Footnote omitted.] The jury

returned a verdict in favor of Plaintiffs and against the City and awarded damages in the

amount of $4.5 million to Adkins and $580,000 to Fernandez.

The trial court molded the verdicts to $442,913.35 for Adkins and $57,086.65 for

Fernandez, and by separate orders awarded delay damages in the amount of $66,303.73 to

Adkins and $3,048.34 to Fernandez. Plaintiffs filed amended motions for delay damages

and the City filed a motion for post-trial relief seeking judgment notwithstanding the

verdict or a new trial. The trial court granted Plaintiffs' motions and amended Adkins'

award of delay damages to $673,642.50 and Fernandez' award to $30,319.14. The court

denied the City's motion for post-trial relief.

***

The City argues that 42 Pa.C.S. § 8553(d) requires the court to subtract any applicable

insurance proceeds from the maximum statutory recovery of $500,000 as the legislature

attempted to limit the exposure of municipal governments and to allow those same bodies

to offset insurance proceeds; and the trial court did not have jurisdiction to revisit the

issue of delay damages because the court ordered those damages five months earlier and

could not amend its order after thirty days from the date it was issued.

The extent to which damage awards are recoverable from a municipality is set forth in 42

Pa.C.S. § 8553, which provides in pertinent part:

67

Page 72: OCTOBER 2020 PENNSYLVANIA BAR EXAMINATIONQuestion No. 1 Susan and Doug are a married couple who reside in Steel Town, Pennsylvania. They have been married for 25 years and have one

(b) Amounts recoverable. — Damages arising from the same cause of

action or transaction or occurrence or series of causes of action or

transactions or occurrences shall not exceed $500,000 in the aggregate.

(c) Types of losses recognized. — Damages shall be recoverable only for:

(1) Past and future loss of earnings and earning capacity.

(2) Pain and suffering in the following instances:

(i) death; or

(ii) only in cases of permanent loss of a bodily function,

permanent disfigurement or permanent dismemberment where

the medical and dental expenses referred to in paragraph (3) are

in excess of $1,500.

(3) Medical and dental expenses including the reasonable value of

reasonable and necessary medical and dental services, prosthetic

devices and necessary ambulance, hospital, professional nursing, and

physical therapy expenses accrued and anticipated in the diagnosis,

care and recovery of the claimant.

(4) Loss of consortium.

(5) Loss of support.

(6) Property losses.

(d) Insurance benefits. — If a claimant receives or is entitled to receive

benefits under a policy of insurance other than a life insurance policy as a

result of losses for which damages are recoverable under subsection (c), the

amount of such benefits shall be deducted from the amount of damages which

would otherwise be recoverable by such claimant. [Emphasis added.]

***

[I]t is axiomatic that where language in a statute is explicit and clear, this Court will not

disturb the plain meaning of that language by resorting to the rules of statutory

construction. [Citation omitted.] While 1 Pa.C.S. § 1924 permits a court to consider the

title of a statute when it seeks to ascertain and effectuate the intention of the General

Assembly, it expressly prohibits considering "headings prefixed to titles, parts, articles,

chapters, sections and other divisions of a statute" as controlling over the statute's plain

words. [Citation omitted.]

The plain words in Section 8553 provide that insurance benefits should be deducted from a

jury verdict before the verdict is molded to the statutory limit. Section 8553(d) specifically

refers to damages which are recoverable under Section 8553(c), and subsection (d) makes

no reference to Section 8553(b) which specifies the $500,000 statutory limit. Those

damages which would otherwise be recoverable and from which insurance benefits shall be

deducted are specified in subsection (c) and include, inter alia, damages recoverable in

customary tort claims for lost wages, medical expenses, and pain and suffering. The jury

awards represent the damages recoverable by Fernandez and Adkins under subsection (c).

Thus any medical insurance benefits received by Adkins should be deducted from the jury

award in his favor, not the statutory cap.

68

Page 73: OCTOBER 2020 PENNSYLVANIA BAR EXAMINATIONQuestion No. 1 Susan and Doug are a married couple who reside in Steel Town, Pennsylvania. They have been married for 25 years and have one

Had the legislature intended the result urged by the City . . . , the legislature would have

specifically provided that insurance benefits shall be deducted from the molded statutory

cap of $500,000 in accordance with subsection (b) entitled "Amounts recoverable" rather

than specifically and exclusively referring in subsection (d) to damages which are

otherwise recoverable under subsection (c). [Footnote omitted.] The trial court therefore

did not err by deducting Adkins' insurance proceeds from the jury verdict. . . . Accordingly,

based upon the foregoing reasons, the order[] of the trial court [is] affirmed.

***

[Concurring and dissenting opinions omitted]

69

Page 74: OCTOBER 2020 PENNSYLVANIA BAR EXAMINATIONQuestion No. 1 Susan and Doug are a married couple who reside in Steel Town, Pennsylvania. They have been married for 25 years and have one

Instructions

The performance test is designed to test an applicant’s ability to perform the

legal task that has been assigned using the factual information contained in the

File and legal principles that are provided in the Library.

The File contains the only factual information that you should consider in

performing the assigned task. The task to be completed is set forth in the first

document in the File in the form of a memorandum to the applicant. The

Library contains the only legal principles that you should consider to complete

the assigned task. Although your general knowledge of the law may provide

some background for analyzing the problem, the factual information contained

in the File and the legal principles contained in the Library are the only

materials that you should use in formulating your answer to the assigned task.

Your response should be typed in the exemplify screen designated for the PT

answer. Be sure to allow sufficient time for reading the materials, organizing

your answer and completing the task assigned. Your answer should

demonstrate an understanding of the relevant facts, recognition of the issues

and the applicable principles of law and the reasoning that supports your

answer. Your grade will be based on the content of your response and your

ability to follow instructions in performing the assigned task.

The events depicted and the persons portrayed by the information in the File

are fictitious and such information does not depict nor is it intended to depict

or portray any actual person, company or occurrence. Any similarity to any

person, living or dead, or any occurrence is purely coincidental.

70

Page 75: OCTOBER 2020 PENNSYLVANIA BAR EXAMINATIONQuestion No. 1 Susan and Doug are a married couple who reside in Steel Town, Pennsylvania. They have been married for 25 years and have one

Question No. PT: Examiner’s Analysis

The applicant is assigned to draft an Internal Legal Memorandum addressed to the assigning Managing

Partner providing a legal analysis concerning whether the client can bring an action for the intentional

tort of conversion against a school district and school district superintendent, in his official capacity, and

related matters including sovereign immunity and a cap on damages.

Formatting 1 Point

Following directions is an important skill of every lawyer. The applicant is expected to present the

analysis in the form of an internal legal memorandum that addresses the issues identified.

Conversion 4 Points

A [common law] conversion is the deprivation of another's right of property in, or use or possession of,

a chattel, or other interference therewith, without the owner's consent and without lawful justification.

Stevenson v. Economy Bank of Ambridge.

Conversion is an act of interference with the dominion or control over a chattel. Stevenson v. Economy

Bank of Ambridge.

Conversion may be committed by unreasonably withholding possession from one who has the right to it.

Stevenson v. Economy Bank of Ambridge.

Superintendent Tierney took possession of the watch from Joseph Anderson in his presence and with his

acquiescence. Intake Memorandum.

There is no dispute concerning who actually owns the watch; it is owned by Luke Anderson.

Superintendent Tierney’s e-mail message confirms this. Intake Memorandum.

Luke Anderson won the watch; he purchased an insurance policy on the watch; took steps to safeguard

his possession of the watch; and sought the return of the watch from Superintendent Tierney. Intake

Memorandum.

Like the Stevenson case, where the bank delayed access to that claimant’s funds but did not take those

funds, Superintendent Tierney has said the watch can be retrieved when environmental consultants say it

is safe to return to the building; he has not yet said Mr. Anderson will never get possession of the watch

again, although he indicated it may never be safe to enter the building again. Stevenson v. Economy

Bank of Ambridge and Intake Memorandum.

It appears that the initial action by Superintendent Tierney in taking the watch from Joseph Anderson

was reasonable under the circumstances. Joseph and another student were fighting over the watch,

which was valued at $750,000, and Superintendent Tierney placed the watch in his own desk for

safekeeping until Joseph was ready to go home. Intake Memorandum. Therefore, the conduct that is the

subject of the conversion would be Superintendent Tierney’s closing of the school while the watch was

71

Page 76: OCTOBER 2020 PENNSYLVANIA BAR EXAMINATIONQuestion No. 1 Susan and Doug are a married couple who reside in Steel Town, Pennsylvania. They have been married for 25 years and have one

still inside and not permitting Mr. Anderson to enter and retrieve his watch, thereby depriving Mr.

Anderson of his use and possession of the watch.

Despite the potential for Mr. Anderson to someday retrieve the watch, Superintendent Tierney is

presently depriving Mr. Anderson of his use and possession of his watch. Intake Memorandum.

Whether because of an environmental issue or under Mr. Anderson’s premise that he wants to keep the

watch for himself, Superintendent Tierney has deprived Mr. Anderson of his property. Intake

Memorandum.

The watch is clearly being held against Mr. Anderson’s wishes as Mr. Anderson has repeatedly

demanded its return and Superintendent Tierney has refused to return Mr. Anderson’s calls and most e-

mail messages. Thus, Superintendent Tierney has denied Mr. Anderson access to the watch. Intake

Memorandum.

Superintendent Tierney may argue that he is justified in refusing access to the building for

environmental considerations and, thus, claim that he is lawfully justified in denying Mr. Anderson

access to the watch. However, Superintendent Tierney allowed students, faculty, and even himself to

work and attend school in the building despite the presence of asbestos containing materials (ACM) in

the building, instead indicating that it was safe to occupy the building until remediation could be done

during the winter break. Nothing in the condition of the building changed since early July 2020 that

would render it immediately unsafe; rather the change was in Superintendent Tierney’s understanding of

the severity of the environmental situation. Further, Mr. Anderson has offered to hire an environmental

technician of the school district’s choosing to wear proper protective equipment to enter into the school

to retrieve the watch. Thus, Mr. Anderson has an argument that there is no lawful justification for

keeping the watch at the school. Intake Memorandum.

If such an argument is accepted, then Superintendent Tierney is interfering with Mr. Anderson’s

dominion and control over the watch. Stevenson v. Economy Bank of Ambridge and Intake

Memorandum.

Mr. Anderson can also argue that Superintendent Tierney is unreasonably withholding possession of the

watch from Mr. Anderson, the rightful owner of the watch. Even if there is a legitimate environmental

issue in the school, Mr. Anderson has offered to pay for an environmental technician of the school’s

choosing to don the proper equipment to go in and retrieve the watch, but Superintendent Tierney has

not allowed him to do so. It seems patently unreasonable to refuse access to such a valuable item

indefinitely under the circumstances. Stevenson v. Economy Bank of Ambridge and Intake

Memorandum.

Superintendent Tierney asserts that allowing the technician in could expose the school district to

additional liability from students or teachers who later claim something is missing and invite claims of

unfair treatment. Intake Memorandum. Nonetheless, it is not likely that other students would have

items valued at $750,000 stored at the school. Thus, there may be a legitimate basis to treat Mr.

Anderson’s watch differently.

72

Page 77: OCTOBER 2020 PENNSYLVANIA BAR EXAMINATIONQuestion No. 1 Susan and Doug are a married couple who reside in Steel Town, Pennsylvania. They have been married for 25 years and have one

From the facts and caselaw it appears that Mr. Anderson may be able to make out a prima facie claim of

conversion. Stevenson v. Economy Bank of Ambridge and Intake Memorandum.

Alternatively, if Superintendent Tierney’s position is accepted, that no one should be allowed in the

building because of the asbestos contamination, and that there could be legitimate claims brought

against the school district for selectively allowing an environmental technician in to retrieve the property

of one student, then Superintendent Tierney may have lawful justification to withhold possession of the

watch from Mr. Anderson, as least for some period of time.

School District Immunity 5 Points

A “Local Agency” is defined as a “government unit other than the Commonwealth government. The

term includes . . . [a school district].” 42 Pa. C.S. § 8501.

Midtown School District is a school district pursuant to the definition of 42 Pa. C.S. § 8501, meaning it

is subject to the applicable statutes contained in the Library. 42 Pa. C.S. § 8501.

Except as otherwise provided in this subchapter, no local agency shall be liable for any damages on

account of any injury to a person or property caused by any act of the local agency or an employee

thereof or any other person. 42 Pa. C.S. § 8541.

As a general rule, Midtown School District is not liable for damages to a person or property. 42 Pa. C.S.

§ 8541.

Exceptions to the governmental immunity are set forth in the statute. 42 Pa. C.S. § 8541.

Liability may be imposed upon a school district “on account of an injury to person or property if the

damages would have been recoverable under common law, if the injury was not caused by a person with

governmental immunity or official immunity, and the injury was caused by the negligent acts of the

local agency or an employee thereof acting within the scope of his duties. Negligent acts shall not

include acts or conduct that constitutes a crime, actual fraud, [or] actual malice. . . .” 42 Pa. C.S. §§

8542(a)(1) and (2).

Specifically, if 42 Pa. C.S. §§ 8542(a)(1) and (2) are met, 42 Pa. C.S. § 8542(b)(2) states the district

shall be liable for injury to property if the personal property of others is in the care, custody or control of

property in possession of the local agency, where property losses are suffered with respect to the

property in the possession or control of the local agency. 42 Pa. C.S. § 8542(b)(2).

Superintendent Tierney took the watch into his and the school district’s care, custody and control when

he took possession of the watch from Joseph Anderson. Intake Memorandum.

However, the requirements of 42 Pa. C.S. § 8542(a)(1) and (2) are not met because Superintendent

Tierney has official immunity (see below), and because the injury was likely not caused by a negligent

act; rather Superintendent Tierney’s acts with regard to taking the watch and keeping the school closed

73

Page 78: OCTOBER 2020 PENNSYLVANIA BAR EXAMINATIONQuestion No. 1 Susan and Doug are a married couple who reside in Steel Town, Pennsylvania. They have been married for 25 years and have one

were intentional (Assignment Memorandum). Thus, Midtown School District should not be held liable

for Mr. Anderson’s property loss even though the watch remains in the care, custody, and control of the

school district, as the second element contained in 42 Pa.C.S.§ 8542(a) is not met. 42 Pa. C.S. §

8542(b)(2).

Superintendent Tierney’s Immunity 7 Points

An employee of a local agency is liable for civil damages on account of any injury to a person or

property caused by acts of the employee which are within the scope of his office or duties only to the

same extent as his employing local agency and subject to the limitations imposed by this subchapter. 42

Pa. C.S. § 8545.

Superintendent Tierney may, generally, be held liable to the same extent Midtown School District may

be held liable for the acts of Superintendent Tierney, its employee. 42 Pa. C.S. § 8545.

In any action brought against an employee of a local agency for damages on account of an injury to a

person or property based upon claims arising from, or reasonably related to, the office or the

performance of the duties of the employee, the employee may assert on his own behalf, or the local

agency may assert on his behalf:

(1) Defenses which are available at common law to the employee.

(2) The defense that the conduct of the employee which gave rise to the claim was authorized or

required by law, or that he in good faith reasonably believed the conduct was authorized or

required by law.

42 Pa.C.S. § 8546.

Superintendent Tierney may assert a defense of official immunity if a common law defense is available,

or if his conduct was authorized or required by law, or if he in good faith reasonably believed the

conduct was authorized or required by law. 42 Pa.C.S. § 8546.

Superintendent Tierney may have a common law defense, as he believed the act of closing the school

and refusing to allow others in was necessary for the health and safety of the students and faculty, and

conversion requires the absence of lawful justification when depriving another of his right to use or

possess property. Intake Memorandum and Stevenson v. Economy Bank of Ambridge.

Further, conversion requires the unreasonable withholding of possession from one who has the right to

it. Stevenson v. Economy Bank of Ambridge.

If Superintendent Tierney’s closing of the school, while the watch remained inside, for the health and

safety of the students and faculty is a reasonable action, he would prevail against Mr. Anderson’s claim

of conversion, and his assertion of official immunity would be upheld under this common law defense.

42 Pa.C.S. § 8546(1).

74

Page 79: OCTOBER 2020 PENNSYLVANIA BAR EXAMINATIONQuestion No. 1 Susan and Doug are a married couple who reside in Steel Town, Pennsylvania. They have been married for 25 years and have one

Although Mr. Anderson has offered to hire an environmental technician who would wear appropriate

protective gear to enter the school and retrieve the watch, Superintendent Tierney may properly object to

that if he reasonably believes that it is not safe for anyone to enter the school. This is demonstrated by

his refusal to permit anyone to enter the school to retrieve personal belongings. Superintendent Tierney

also may have an additional reasonable basis for refusing to permit Mr. Anderson’s environmental

technician to enter the building, as it may expose the district to additional liability from a student or

teacher who claims something is missing after the consultant entered the building, or to claims of unfair

treatment by allowing one person to retrieve his belongings, but not all others. Intake Memorandum.

Superintendent Tierney may also have a defense because his conduct that gave rise to the claim was

authorized or required by law, or that he in good faith reasonably believed the conduct was authorized or

required by law. 42 Pa.C.S. § 8546(2).

In television interviews, Superintendent Tierney cited a statute providing for the closure of the school if

it was for the health and safety of the students and faculty. He further cited to his interpretation of

federal environmental statutes as the basis for concluding that the school was no longer safe. Intake

Memorandum. The materials provide no information concerning the validity of these statutes or an

interpretation of the federal statutes that would be contrary to Superintendent Tierney’s interpretation, so

we can only conclude that he believed he was authorized or required by law to close the school.

The materials also provide little information that would allow an applicant to conclude that

Superintendent Tierney was acting in bad faith in closing the schools. Superintendent Tierney stated he

was closing the schools for the health and safety of the faculty; that he was statutorily authorized to do

so; that he was encouraged by the school district’s environmental consultant to close the school; and that

his basis for closing the school now was an admitted mistake in his reading of the environmental reports

that clearly demonstrated a presence of potentially harmful asbestos in the school. Intake Memorandum.

Mr. Anderson has speculated that Superintendent Tierney closed the school to keep the watch (Intake

Memorandum); however, there are few facts in the materials that would allow an applicant to conclude

the closing of the school was for an improper purpose. These include:

1. A friend of Mr. Anderson said he saw something shiny on Superintendent’s wrist while at

dinner one night; however, there are no facts in the materials that demonstrate the shiny

object on Superintendent Tierney’s wrist was the watch in question, as opposed to another

watch or something entirely different. Intake Memorandum.

2. Superintendent Tierney was also overheard saying “[t]his is a gorgeous watch. I wish I

had won it.” Intake Memorandum. This could be interpreted as having ill-intent, or

simply as a statement that nearly anyone would make with regard to winning an

item worth $750,000.

3. Superintendent Tierney closed the school with no notice, reversing course on his

earlier decision to keep the school open, on the same day he took possession of

the watch. Intake Memorandum. This change in position could have been done

in an attempt to keep the watch, but Superintendent Tierney explained it was

75

Page 80: OCTOBER 2020 PENNSYLVANIA BAR EXAMINATIONQuestion No. 1 Susan and Doug are a married couple who reside in Steel Town, Pennsylvania. They have been married for 25 years and have one

done as a result of a telephone call from an environmental consultant, and his

misunderstanding of the significance of the asbestos contamination.

In order for Mr. Anderson to prevail on a claim against Superintendent Tierney, he would have to show

that the actions taken by Superintendent Tierney were unreasonable, were not authorized by statute, and

were not done in good faith. As there are few facts in the materials that would provide support for such

a proposition, Mr. Anderson’s conversion claim against Superintendent Tierney would likely be barred

by official immunity.

Accepting the proposition that Superintendent Tierney intends to keep the watch for himself would

require accepting that he closed a school; displaced 900 students plus faculty and staff; stole a highly

valuable watch that is the subject of news stories and publicity; publicly wore the watch despite claiming

it was locked in his office; publicly admitted that he made a mistake that could affect the health of 900

students, faculty and staff in an effort to conceal his alleged misdeed; and thought he could get away

unscathed. Intake Memorandum.

Further, there are no facts or law that suggest Superintendent Tierney’s acts would be judicially

determined to constitute a crime, actual fraud, [or] actual malice . . . that would enable Mr. Anderson to

ask a court to set aside official immunity under 42 Pa. C.S. § 8550. Applicants are instructed to rely

only upon the materials provided, and no materials setting forth the elements of a crime are contained in

the Library or File.

In the event applicants conclude that it would be judicially determined that Superintendent Tierney has

committed a crime, actual fraud, or actual malice, then under 42 Pa. C.S. §8550, the provisions

pertaining to official immunity shall not apply.

Limitation on Damages 3 Points

42 Pa. C.S. § 8549 provides:

In any action brought against an employee of a local agency for damages on account of an injury to a

person or property in which it is judicially determined that the act of the employee caused the injury and

that such act was, or that the employee in good faith reasonably believed that such act was, within the

scope of his office or duties, damages shall be recoverable only within the limits set forth in this

subchapter.

Damages from any action brought for property damages against Superintendent Tierney, an employee of

the school district (Intake Memorandum) for his conduct in closing the school, which he claims to have

done for the health and safety of the students and faculty (Intake Memorandum) will only be recoverable

within the limits of the official immunity statute. 42 Pa. C.S. § 8549.

42 Pa. C.S. § 8553 provides:

***

76

Page 81: OCTOBER 2020 PENNSYLVANIA BAR EXAMINATIONQuestion No. 1 Susan and Doug are a married couple who reside in Steel Town, Pennsylvania. They have been married for 25 years and have one

(b) Amounts recoverable. Damages arising from the same cause of action or transaction or

occurrence or series of causes of action or transactions or occurrences shall not exceed $500,000

in the aggregate.

(c) Types of losses recognized. Damages shall be recoverable only for:

***

(6) Property losses.

(d) Insurance benefits. If a claimant receives or is entitled to receive benefits under a policy of

insurance other than a life insurance policy as a result of losses for which damages are

recoverable under subsection (c), the amount of such benefits shall be deducted from the amount

of damages which would otherwise be recoverable by such claimant.

Mr. Anderson can make a claim for property losses (42 Pa. C.S. § 8553(c)), but his actual recovery, if he

were to prevail, would be limited to Five Hundred Thousand Dollars ($500,000) instead of his claimed

damages totaling the full value of the watch, which is Seven Hundred Fifty Thousand Dollars

($750,000). 42 Pa. C.S. § 8553(b).

Further, Mr. Anderson’s claim would have to take into account the insurance policy he purchased on the

watch.

“The plain words in Section 8553 provide that insurance benefits should be deducted from a jury verdict

before the verdict is molded to the statutory limit.” Fernandez v. City of Pittsburgh.

If Mr. Anderson were to prevail, his requested damages would be $750,000, or the full value of the

watch. Intake Memorandum.

Mr. Anderson took out a policy of insurance for One Hundred Thousand Dollars ($100,000) on the

watch. Intake Memorandum.

While Mr. Anderson’s recoverable damages would be reduced by the amount of insurance available to

him (42 Pa. C.S. § 8553(d)), this would not impact Mr. Anderson’s actual recovery, as it would reduce

his total damages of $750,000 by the insurance proceeds of $100,000, leaving him a recovery of

$650,000; this amount, however, is more than the amount recoverable of $500,000 (42 Pa.C.S. §

8553(b)); and so any verdict or award would be reduced to $500,000, if Mr. Anderson were to prevail

against the school district or the district superintendent acting in his official capacity.

77